Крок 1 - Медицина 2014 (буклет)

1 / 200
У сироватці крові пацієнта встановлено підвищення активності гіалуронідази. Визначення якого біохімічного показника сироватки крові дозволить підтвердити припущення про патологію сполучної тканини? Increased hyaluronidase activity was detected in the patient's blood serum. The determination of which biochemical indicator of the blood serum will confirm the hypothesis of connective tissue pathology?

Галактоза Galactose

Білірубін Bilirubin

Сечова кислота Uric acid

Глюкоза Glucose

Сіалові кислоти Sialic acids

2 / 200
При мікроскопії мікропрепарату з виділень хворої хронічним кольповагінітом лікар виявив округлої форми та еліпсоподібні клітини, що брунькуються, розміром 3-6 мкм. Про збудника якої грибкової хвороби може йти мова в даному випадку? During microscopy of a micropreparation from secretions of a patient with chronic colpovaginitis, the doctor found rounded and elliptical budding cells, 3-6 microns in size. What fungal disease can we be talking about in this case?

Мікроспорія Microsporia

Епідермофітія Epidermophytia

Кандидоз Candidiasis

Криптококоз Cryptokokokot

Кокцидіоз Coccidiosis

3 / 200
У пацієнта з ознаками коліту виділена чиста культура бактерій, яка за морфологічними, культуральними та біохімічними властивостями віднесена до роду шигел. Яку з названих реакцій доцільно застосувати для серологічної ідентифікації культури? A pure culture of bacteria was isolated from a patient with signs of colitis, which according to morphological, cultural and biochemical properties is classified as Shigella. Which of the above reactions should be used for serological identification of the culture ?

Затримки гемаглютинації Hemagglutination delays

Аглютинації з діагностичними сироватками Agglutinations with diagnostic sera

Преципітації Precipitation

Зв’язування комплементу Complement Binding

Непрямої гемаглютинації Indirect hemagglutination

4 / 200
У пацієнтки 20-ти років встановлено діагноз - СНІД. Які популяції клітин найбільш чутливі до вірусу імунодефіциту людини? A 20-year-old female patient has been diagnosed with AIDS. Which cell populations are most sensitive to human immunodeficiency virus?

Т-хелпери T Helper

Гепатоцити Hepatocytes

В-лімфоцити B-lymphocytes

Ендотеліоцити Endotheliocytes

Епітеліоцити Epitheliocytes

5 / 200
Після загоєння рани на її місці утворився рубець. Яка речовина є основним компонентом цього різновиду сполучної тканини? After the wound healed, a scar formed in its place. What substance is the main component of this type of connective tissue?

Еластин Elastin

Колаген Колаген

Гіалуронова кислота Hyaluronic acid

Кератансульфат Keratan Sulfate

Хондроітин-сульфат Chondroitin Sulfate

6 / 200
У спортсмена внаслідок довільної затримки дихання на 40 секунд зросли частота серцевих скорочень та системний артеріальний тиск. Реалізація яких механізмів регуляції зумовлює зміни показників? As a result of an arbitrary breath hold for 40 seconds, the athlete's heart rate and systemic blood pressure increased. The implementation of which regulatory mechanisms causes changes in indicators?

Умовні симпатичні рефлекси Conditional sympathetic reflexes

Безумовні симпатичні рефлекси Unconditioned sympathetic reflexes

Безумовні парасимпатичні рефлекси Unconditioned parasympathetic reflexes

- -

Умовні парасимпатичні рефлекси Conditional parasympathetic reflexes

7 / 200
У малярійного плазмодія - збудника триденної малярії, розрізняють два штами: південний та північний. Вони відрізняються тривалістю інкубаційного періоду: у південного він короткий, а у північного - довгий. В цьому проявляється виражена дія такого добору: There are two strains of malaria plasmodium, the causative agent of three-day malaria: southern and northern. They differ in the length of the incubation period: the southern one has a short incubation period, and the northern one has a long one. This shows the pronounced effect of such selection:

Штучний Artificial

Рушійний Driver

Стабілізуючий Stabilizing

Статевий Gender

Дизруптивний Disruptive

8 / 200
Струми надвисокої частоти (НВЧ), що застосовуються у фізіотерапії, не викликають збудження, а зумовлюють лише тепловий ефект на тканини. Як можна пояснити це явище? Ultra-high frequency (UHF) currents used in physiotherapy do not cause excitement, but cause only a thermal effect on tissues. How can this phenomenon be explained?

Тривалість стимулу менше порога The duration of the incentive is less than the threshold

Стимул поступає у фазу абсолютної рефрактерності The stimulus enters the phase of absolute refractoriness

Стимул поступає у фазу відносної рефрактерності The stimulus enters the phase of relative refractoriness

Розвивається акомодація Developing accommodation

Інтенсивність стимулу менше порога Stimulus intensity less than threshold

9 / 200
У чоловіка 35-ти років феохромоцитома. В крові виявляється підвищений рівень адреналіну та норадреналіну, концентрація вільних жирних кислот зросла в 11 разів. Активація якого ферменту під впливом адреналіну підвищує ліполіз? A 35-year-old man has a pheochromocytoma. An elevated level of adrenaline and noradrenaline is detected in the blood, the concentration of free fatty acids has increased 11 times. The activation of which enzyme under the influence of adrenaline increases lipolysis?

Фосфоліпаза С Phospholipase C

ТАГ-ліпаза TAG-lipase

Фосфоліпаза А2 Phospholipase A2

Холестеролестераза Cholesterolesterase

Ліпопротеїдліпаза Lipoprotein lipase

10 / 200
В результаті травми пошкоджений спинний мозок (з повним розривом) на рівні першого шийного хребця. Що відбудеться з диханням? As a result of the injury, the spinal cord is damaged (with a complete rupture) at the level of the first cervical vertebra. What will happen to breathing?

Зростає частота Increasing frequency

Зменшується частота Decreasing frequency

Припиняється Stopping

Не змінюється Does not change

Зростає глибина Increasing depth

11 / 200
У людини з хронічним захворюванням нирок порушена їх видільна функція. рН венозної крові становить 7,33. Для корекції кислотно-лужного стану пацієнту доцільно внутрішньовенно ввести розчин: In a person with chronic kidney disease, their excretory function is impaired. The pH of venous blood is 7.33. To correct the acid-alkaline state, it is advisable to inject a solution into the patient intravenously:

Глюкози Glucose

Хлориду калію Potassium chloride

Хлориду натрію Sodium Chloride

Бікарбонату натрію Sodium bicarbonate

Хлориду кальцію Calcium chloride

12 / 200
При обтураційній жовтяниці і жовчних норицях часто спостерігається протромбінова недостатність. З дефіцитом в організмі якого вітаміну це пов’язано? Prothrombin deficiency is often observed in obstructive jaundice and biliary fistulas. What vitamin deficiency is this associated with in the body?

C C

В6 В6

E E

K K

A A

13 / 200
При розтині тіла померлого виявлена гіперплазія кісткового мозку плоских і трубчастих кісток (піоїдний кістковий мозок), спленомегалія (6 кг), гепатомегалія (5 кг), збільшення всіх груп лімфатичних вузлів. Якому захворюванню відповідають виявлені зміни? During the autopsy of the deceased, hyperplasia of the bone marrow of flat and tubular bones (pyoid bone marrow), splenomegaly (6 kg), hepatomegaly (5 kg), an increase in all groups was found lymph nodes. What disease do the detected changes correspond to?

Справжня поліцитемія Polycythemia True

Хронічний мієлолейкоз Chronic myelogenous leukemia

Лімфогрануломатоз Lymphogranulomatosis

Мієломна хвороба Myeloma

Хронічний лімфолейкоз Chronic lymphocytic leukemia

14 / 200
У молодої жінки в зв’язку з гострими болями в клубовій ділянці, вилучена маткова труба з локальним розширенням її середньої третини, що заповнено кров’ю. При гістологічному дослідженні в отворі труби знайдені хоріальні ворсини, великі поля еритроцитів з домішком лейкоцитів. Який найбільш імовірний діагноз? A young woman had a fallopian tube removed with a local expansion of its middle third, which was filled with blood, due to acute pain in the iliac region. During histological examination chorionic villi, large fields of erythrocytes with an admixture of leukocytes were found in the opening of the tube. What is the most likely diagnosis?

Геморагічний сальпінгіт Hemorrhagic salpingitis

Гострий гнійний сальпінгіт Acute purulent salpingitis

Крововилив в маткову трубу Bleeding in the fallopian tube

Трубна вагітність Tapal pregnancy

Гнійний сальпінгіт Suppurative salpingitis

15 / 200
При дослідженні крові хворого виявлено значне збільшення активності МВ-форм КФК (креатинфосфокінази) та ЛДГ-1. Яку патологію можна припустити? When examining the patient's blood, a significant increase in the activity of MV-forms of CK (creatine phosphokinase) and LDH-1 was detected. What pathology can be assumed?

Гепатит Hepatitis

Холецистит Cholecystitis

Панкреатит Pancreatitis

Ревматизм Rheumatism

Інфаркт міокарда Myocardial infarction

16 / 200
До лікарні надійшла дитина з діагнозом 'стафілококовий сепсис'. На яке живильне середовище потрібно посіяти кров хворого з метою виділення збудника? A child was admitted to the hospital with a diagnosis of 'staphylococcal sepsis'. On which nutrient medium should the patient's blood be inoculated in order to isolate the pathogen?

Середовище Плоскірьова Ploskiryov Environment

М ’ясо-пептонний агар Meat-peptone agar

Середовище Бучіна Buchin Environment

Цукрово-пептонний бульйон Sugar-peptone broth

Жовчно-сольовий агар Bile-salt agar

17 / 200
У чоловіка 62-х років видалено нирку, у якій при макроскопічному дослідженні виявлено пухлину у вигляді вузла діаметром до 8 см. Тканина пухлини на розрізі строката, з множинними крововиливами, некрозами. Гістологічно: пухлина складається із світлих клітин, які утворюють альвеолярні і сосочкові структури, помірно виражений інвазивний ріст. У багатьох клітинах пухлини визначаються патологічні мітози, гіперхромні ядра. Діагностуйте виявлену пухлину нирки: A 62-year-old man had his kidney removed, in which a macroscopic examination revealed a tumor in the form of a nodule with a diameter of up to 8 cm. The tumor tissue on the section is variegated, with multiple hemorrhages , necrosis. Histologically: the tumor consists of light cells that form alveolar and papillary structures, moderately pronounced invasive growth. Pathological mitoses, hyperchromic nuclei are detected in many tumor cells. Diagnose the detected kidney tumor:

Аденокарцинома Adenocarcinoma

Нефробластома Nephroblastoma

Світлоклітинний рак Clear cell carcinoma

Світлоклітинна аденома Clear cell adenoma

Ацидофільна аденома з малігнізацією Acidophilic adenoma with malignancy

18 / 200
Безпосередньо після переходу з горизонтального положення у вертикальне у чоловіка частота серцевих скорочень збільшилась на 15 скорочень за хвилину. Які механізми регуляції переважно зумовлюють цю зміну? Immediately after the transition from a horizontal position to a vertical position, the heart rate of a man increased by 15 beats per minute. What regulatory mechanisms mainly cause this change?

Умовні та безумовні симпатичні рефлекси Conditional and unconditional sympathetic reflexes

Катехоламіни Catecholamines

Симпатичні рефлекси і катехоламіни Sympathetic reflexes and catecholamines

Безумовні симпатичні рефлекси Unconditioned sympathetic reflexes

Умовні симпатичт рефлекси Conditional sympathetic reflexes

19 / 200
У людини з масою 80 кг після тривалого фізичного навантаження об’єм циркулюючої крові зменшився, гематокрит - 50%, загальний білок крові - 80 г/л. Такі показники крові є наслідком, перш за все: In a person with a mass of 80 kg, after prolonged physical exertion, the volume of circulating blood decreased, hematocrit - 50%, total blood protein - 80 g/l. Such indicators of blood is a consequence, first of all:

Збільшення онкотичного тиску плазми Increase in oncotic plasma pressure

Втрати води з потом Sweat water loss

Збільшення кількості еритроцитів Increase in the number of erythrocytes

Збільшення вмісту білків у плазмі Increase in plasma proteins

Збільшення діурезу Increased diuresis

20 / 200
Після перенесеного запального процесу хворий став помічати слабкість при згинанні кисті в ділянці 1,2, 3 і 4 пальців, зменшення об’єму м’яза підвищення великого пальця. При обстеженні виявлено порушення больової і температурної чутливості в ділянці поверхні долоні 1, 2, 3 і променевої поверхні четвертого пальців. Який із нервів уражений? After the inflammatory process, the patient began to notice weakness when bending the hand in the area of 1, 2, 3 and 4 fingers, a decrease in the volume of the thumb muscle. During the examination, a violation of pain and temperature sensitivity was revealed in the area of the surface of the palm 1, 2, 3 and the radial surface of the fourth fingers. Which of the nerves is affected?

Променевий Radial

М’язово-шкірний Musculocutaneous

Ліктьовий Cubit

Присередній шкірний нерв передпліччя Median cutaneous nerve of the forearm

Серединний Middle

21 / 200
У чоловіка 53-х років діагностовано сечокам’яну хворобу з утворенням уратів. Цьому пацієнту призначено аллопурінол, який є конкурентним інгібітором ферменту: A 53-year-old man was diagnosed with urolithiasis with urate formation. This patient was prescribed allopurinol, which is a competitive enzyme inhibitor:

Дигідроурацилдегідрогеназа Dihydrouracil dehydrogenase

Ксантиноксидаза Xanthine Oxidase

Уратоксидаза Urate Oxidase

Уриділтрансфераза Uridyltransferase

Уреаза Уреаза

22 / 200
В експерименті на тварині досліджують серцевий цикл. Закриті усі клапани серця. Якій фазі циклу відповідає такий стан? In an experiment on an animal, the heart cycle is studied. All the heart valves are closed. What phase of the cycle does this state correspond to?

Протодіастолічний період Protodiastolic period

Асинхронного скорочення Asynchronous reduction

Ізометричного скорочення Isometric reduction

Швидкого наповнення Fast filling

Повільного наповнення Slow filling

23 / 200
Крива дисоціації оксигемоглобіну зміщена вправо. Які зміни в організмі людини можуть бути причиною цього? The oxyhemoglobin dissociation curve is shifted to the right. What changes in the human body can be the reason for this?

Збільшення концентрації 2,3-дифосфогліцерату в еритроцитах Increased concentration of 2,3-diphosphoglycerate in erythrocytes

Гіпертермія Hyperthermia

Алкалоз Alkalosis

Гіпоксемія Hypoxemia

Гіпокапнія Hypocapnia

24 / 200
При операції правосторонньої лобектомії хірург підійшов до кореня правої легені з метою окремого виділення і обробки його складових. Вкажіть порядок розміщення елементів кореня правої легені з якими лікар зустрінеться при виділенні та обробці в напрямку зверху вниз? During a right-sided lobectomy operation, the surgeon approached the root of the right lung in order to separate and process its components. Specify the order of placement of the elements of the root of the right lung that the doctor will encounter during the extraction and top-down processing?

Бронх, легенева артерія, легеневі вени Bronchu, pulmonary artery, pulmonary veins

Легенева вена, легеневі артерії, бронх Pulmonary vein, pulmonary arteries, bronchus

Бронх, легенева артерія, діафрагмальний нерв Bronchu, pulmonary artery, phrenic nerve

Діафрагмальний нерв, бронх, бронхіальні артерія і вена Phrenic nerve, bronchus, bronchial artery and vein

Легенева артерія, бронх, легеневі вени Pulmonary artery, bronchus, pulmonary veins

25 / 200
Ціаністий калій є отрутою, від якої смерть організму наступає миттєво. На які ферменти в мітохондріях діє ціанистий калій? Potassium cyanide is a poison from which the death of an organism occurs instantly. What enzymes in mitochondria does potassium cyanide act on?

НАД+ - залежні дегідрогенази NAD+ - dependent dehydrogenases

Цитохром Р-450 Cytochrome P-450

Цитохром В5 Cytochrome B5

Флавінові ферменти Flavin enzymes

Цитохромоксидаза (аа3) Cytochrome oxidase (aa3)

26 / 200
При бактеріоскопічному дослідженні носоглоткового слизу дитини 2,5 років, хворої на назофарингіт, виявлені грамнегативні диплококи, схожі за формою на кавові зерна. Які структури організму дитини найбільш імовірно будуть уражені, якщо ці мікроорганізми проникнуть у кров? During the bacterioscopic examination of the nasopharyngeal mucus of a 2.5-year-old child with nasopharyngitis, gram-negative diplococci were found, similar in shape to coffee beans. What structures of the child's body are most likely to be affected if these microorganisms enter the blood?

Лімфатичні вузли Lymph nodes

Ниркові гломерули Kidney glomeruli

Сєрцєві клапани Heart valves

Сечо-статеві шляхи Urinogenital tract

Оболонки мозку Meninges

27 / 200
У хворого під час трахеотомії виникла виражена кровотеча. Яка артерія була травмована під час операції? The patient had severe bleeding during tracheotomy. Which artery was injured during the operation?

A. laryngea inferior A. laryngea inferior

A. thyroidea ima A. thyroidea ima

A. thyroidea inferior A. thyroidea inferior

A. laryngea superior A. laryngea superior

A. thyroidea superior A. thyroidea superior

28 / 200
При різноманітних захворюваннях рівень активних форм кисню різко зростає, що призводить до руйнування клітинних мембран. Для запобігання цьому використовують антиоксиданти. Найпотужнішим природнім антиоксидантом є: In various diseases, the level of reactive oxygen species increases sharply, which leads to the destruction of cell membranes. Antioxidants are used to prevent this. The most powerful natural antioxidant is:

Гліцерол Glycerol

Жирні кислоти Fatty acids

Глюкоза Glucose

Вітамін D Vitamin D

Альфа-токоферол Alpha Tocopherol

29 / 200
При обстеженні чоловіка 40-ка років було встановлено діагноз: гіпохромна анемія. Який препарат треба призначити для лікування? During the examination of a 40-year-old man, a diagnosis was established: hypochromic anemia. What drug should be prescribed for treatment?

Ферковен Ферковен

Гепарин Heparin

Вікасол Вікасол

Ціанокобаламін Cyanocobalamin

Пентоксил Pentoxyl

30 / 200
У неврологічне відділення з приводу мозкового крововиливу поступив хворий 62-х років. Об’єктивно: стан важкий. Спостерігається наростання глибини і частоти дихання, а потім його зменшення до апное, після чого цикл дихальних рухів відновлюється. Який тип дихання у хворого? A 62-year-old patient was admitted to the neurological department due to a cerebral hemorrhage. Objectively: the condition is severe. An increase in the depth and frequency of breathing is observed, and then it decreases to apnea, after which the cycle of respiratory movements is restored. What type of breathing does the patient have?

Гаспінг-дихання Gasping Breath

Апнейстичне Upneustic

Біота Біота

Чейна-Стокса Cheyne-Stokes

Кусмауля Kusmaul

31 / 200
Хворому проведено субтотальну субфасціальну резекцію щитоподібної залози. У післяопераційному періоді тривалий час зберігається охриплість голосу. Який нерв ушкоджено в ході операції? The patient underwent a subtotal subfascial resection of the thyroid gland. In the postoperative period, the hoarseness of the voice persists for a long time. What nerve was damaged during the operation?

Під’язиковий нерв Hypoglossal nerve

Нижньощелепний нерв Mandibular nerve

Язиковий нерв Lingual nerve

Зворотній гортанний нерв Reverse laryngeal nerve

Верхній гортанний нерв Superior laryngeal nerve

32 / 200
Чоловік 53-х років доставлений у стаціонар у непритомному стані. Об’єктивно: шкіра суха, дихання часте поверхневе, запах ацетону відсутній, Ps- 126/хв., АТ- 70/40 мм рт.ст. Вміст глюкози у крові 48 ммоль/л, реакція сечі на ацетон негативна. Для якого із перелічених станів найбільш характерні симптоми у хворого? A 53-year-old man was brought to the hospital in an unconscious state. Objectively: dry skin, frequent shallow breathing, no smell of acetone, Ps- 126/min. , blood pressure - 70/40 mm Hg. The blood glucose content is 48 mmol/l, the urine reaction to acetone is negative. For which of the listed conditions are the most characteristic symptoms in the patient?

Лактацидемічна кома Lactacidemic coma

Гіперкетонемічна кома Hyperketonemic coma

Колапс Collapse

Токсична кома Toxic Coma

Гіперосмолярна кома Hyperosmolar coma

33 / 200
У гістопрепараті представлений паренхіматозний орган, що має кіркову і мозкову речовину. Кіркова утворена тяжами епітеліоцитів, між якими проходять кровоносні капіляри. Тяжі формують три зони. Мозкова речовина складається з хромафіноцитів і венозних синусоїдів. Який орган має дані морфологічні ознаки? The histopreparation shows a parenchymal organ with cortical and medulla. The cortical is formed by strands of epitheliocytes, between which blood capillaries pass. The strands form three zones. The medulla consists of chromaffinocytes and venous sinusoids. Which organ has these morphological features?

Щитоподібна залоза Thyroid

Нирки Kidneys

Тимус Thymus

Лімфатичний вузол Lymph node

Наднирник Adrenal gland

34 / 200
На спеціальному живильному середовищі після посіву виділень гною з уретри виросли ніжні блакитні колонії. При мікроскопії препаратів з них виявлені грамнегативні бобоподібні диплококи. Збудником якої хвороби вони є? On a special nutrient medium, after inoculation of pus secretions from the urethra, delicate blue colonies grew. Gram-negative leguminous diplococci were detected during microscopy of preparations from them. What disease are they the causative agent of?

Гонорея Gonorrhea

Туляремія Tularemia

Меліоїдоз Melioidosis

Хламідіоз Chlamydia

Сифіліс Syphilis

35 / 200
Жінка 68-ми років скаржиться на відсутність рухів у правих руці і нозі. Чотири місяці тому перенесла інсульт. Об’єктивно: рухи в правих кінцівках відсутні, тонус м’язів їх підвищений. Який стан спостерігається у хворої? A 68-year-old woman complains of a lack of movement in her right arm and leg. She had a stroke four months ago. Objectively: there are no movements in her right limbs, the tone of m 'Their muscles are elevated. What condition is observed in the patient?

Тетраплегія Tetraplegia

Моноплегія Monoplegia

Параплегія Paraplegia

- -

Геміплегія Hemiplegia

36 / 200
У пораненого перев’язали плечову артерію в нижній третині плеча. За рахунок яких артерій можливе відновлення кровопостачання передпліччя і кисті? The brachial artery in the lower third of the shoulder was ligated in the injured person. At the expense of which arteries is it possible to restore blood supply to the forearm and hand?

Підлопаткова, передня міжкісткова артерія Subscapular, anterior interosseous artery

Передні і задні огинаючі артерії плеча Anterior and posterior circumflex arteries of the shoulder

Глибока артерія плеча, ліктьовіколатеральні артерії Deep brachial artery, ulnar collateral arteries

Грудонадплечова, задня огинаюча артерія плеча Thoracobrachial, posterior circumflex brachial artery

М’язові гілки плечової артерії Muscular branches of the brachial artery

37 / 200
У стінці бронху при гістологічному дослідженні чітко визначаються залози, хрящові острівці та багаторядний циліндричний миготливий епітелій. Які бронхи досліджують? Glands, cartilaginous islands and multi-rowed cylindrical ciliated epithelium are clearly defined in the wall of the bronchus during histological examination. Which bronchi are examined?

Середні Average

Великі Large

Головні Main

Термінальні бронхіоли Terminal bronchioles

Малі Small

38 / 200
Під час обстеження у хворої встановлене ураження дорсальної частини мосту, порушена функція жування. Ядро якого нерва уражене? During the examination, the patient was diagnosed with a lesion of the dorsal part of the pons, chewing function was impaired. What nerve nucleus is affected?

Рухове ядро трійчастого нерва Motor nucleus of the trigeminal nerve

Подвійне ядро блукаючого нерва Dual nucleus of the vagus nerve

Рухове ядро лицевого нерва Ruchal nucleus of the facial nerve

Ядро під’язикового нерва Nucleus of the hypoglossal nerve

Мостове ядро трійчастого нерва Principal trigeminal nucleus

39 / 200
Глікоген, що надійшов з їжею, гідролізувався у шлунково-кишковому тракті. Який кінцевий продукт утворився в результаті цього процесу? Glycogen that came with food was hydrolyzed in the gastrointestinal tract. What end product was formed as a result of this process?

Глюкоза Глюкоза

Лактоза Lactose

Лактат Lactate

Фруктоза Fructose

Галактоза Galactose

40 / 200
При бактеріологічному дослідженні сечі хворого на пієлонефрит виділені мікроорганізми, що утворюють на м’ясо-пептонному агарі жовто-зелений пігмент і характерний запах. Як вони називаються? During a bacteriological examination of the urine of a patient with pyelonephritis, microorganisms were isolated that form a yellow-green pigment and a characteristic odor on meat-peptone agar. What are they called?

Псевдомонади Pseudomonas

Ешеріхії Escherichia

Клебсієли Klebsiels

Азотобактерії Azotobacteria

Протеї Proteus

41 / 200
У хворої 28-ми років виявлена пухлина яєчника і показана операція по його видаленню. При здійсненні її необхідно розтинати зв’язку, що з’єднує яєчник з маткою. Яку зв’язку повинен перерізати хірург? In a 28-year-old patient, an ovarian tumor was detected and an operation to remove it is indicated. When performing it, it is necessary to dissect the ligament connecting the ovary to the uterus. Which ligament should the surgeon cut?

Lig. Latum uteri Leg. Wide Uterus

Lig. umbilicale laterale Lateral umbilical ligament

Lig. Ovarii proprium Ovarian ligament

Lig. Cardinali Lead Cardinal

Lig. Suspensorium ovarii Ovarian suspensory ligament

42 / 200
Для підвищення результатів спортсмену рекомендували застосовувати препарат, який містить у собі карнітин. Який процес в найбільшому ступені активується карнітином? To improve results, the athlete was recommended to use a drug that contains carnitine. What process is activated to the greatest degree by carnitine?

Синтез кетонових тіл Synthesis of ketone bodies

Тканинне дихання Tissue respiration

Транспорт жирних кислот у мітохондрії Transport of fatty acids in mitochondria

Синтез стероїдних гормонів Synthesis of steroid hormones

Синтез ліпідів Synthesis of lipids

43 / 200
Хвора 26-ти років звернулася до лікаря зі скаргами на появу у випорожненнях білих плоских рухливих утворів, які нагадують локшину. При лабораторному дослідженні виявлені членики з такою характеристикою: довгі, вузькі, з розміщеним поздовжньо каналом матки, яка має 17-35 бічних відгалужень з кожного боку. Який вид гельмінтів паразитує у кишечнику жінки? A 26-year-old patient turned to the doctor with complaints about the appearance of white, flat, mobile formations that resemble noodles in her stools. Laboratory examination revealed joints with the following characteristics: long , narrow, with a longitudinally placed uterine canal, which has 17-35 side branches on each side. What kind of helminths parasitize in the intestines of a woman?

Taeniarhynchus saginatus Taeniarhynchus saginatus

Echinococcus granulosus Echinococcus granulosus

Hymenolepis nana Hymenolepis nana

Diphyllobothrium latum Diphyllobothrium latus

Taenia solium Tape alone

44 / 200
У жінки 30-ти років при гістологічному дослідженні шийного лімфовузла виявлено порушення малюнка внаслідок розростання епітеліоїдних, лімфоїдних клітин і макрофагів з ядрами у вигляді підкови, в центрі деяких скупчень клітин - безструктурні ділянки блідо-рожевого кольору з уламками ядер. Для якого захворювання характерні такі зміни? In a 30-year-old woman, a histological examination of a cervical lymph node revealed a pattern disturbance due to the growth of epithelioid, lymphoid cells and macrophages with horseshoe-shaped nuclei in the center of some clusters of cells - structureless areas of pale pink color with fragments of nuclei. What disease is characterized by such changes?

Метастаз пухлини Tumor metastasis

Лімфогрануломатоз Lymphogranulomatosis

Сифіліс Syphilis

Туберкульоз Tuberculosis

Актиномікоз Actinomycosis

45 / 200
Хворому з прогресуючою м’язовою дистрофією було проведено біохімічне дослідження сечі. Поява якої речовини у великій кількості в сечі може підтвердити захворювання м’язів у даного хворого? A patient with progressive muscular dystrophy underwent a biochemical examination of urine. The appearance of a substance in large quantities in the urine can confirm muscle disease in this patient?

Креатин Креатин

Гіпурова кислота Hypuric Acid

Креатинін Creatinine

Порфірини Porphyrins

Сечовина Urea

46 / 200
При пункційній біопсії в трансплантованій нирці виявлена дифузна інфільтрація строми лімфоцитами, плазмоцитами, лімфобластами, плазмобластами, а також некротичний артеріїт. Який патологічний процес розвинувся у трансплантаті? Diffuse infiltration of the stroma by lymphocytes, plasma cells, lymphoblasts, plasmablasts, as well as necrotic arteritis was detected in the puncture biopsy in the transplanted kidney. What pathological process developed in the transplant?

Ішемічне пошкодження нирки Ischemic kidney damage

Тубулонекроз Tubulonecrosis

Пієлонефрит Pyelonephritis

Імунне відторгнення Immune rejection

Гломерулонефрит Glomerulonephritis

47 / 200
Який механізм тепловіддачі найбільш ефективно спрацьовує при перебуванні людини в умовах 80% вологості повітря та температурі навколишнього середовища +35oC? Which heat transfer mechanism works most effectively when a person is in conditions of 80% air humidity and an ambient temperature of +35oC?

Потовиділення Sweating

Конвекція Convection

- -

Теплопровідність Thermal conductivity

Радіація Radiation

48 / 200
При обстеженні чоловіка 45-ти років, що тривалий час перебував на рослинній дієті, виявлено негативний азотистий баланс. Яка особливість раціону стала причиною цього явища? During the examination of a 45-year-old man who had been on a plant-based diet for a long time, a negative nitrogen balance was found. What feature of the diet caused this phenomenon?

Надмірна кількість води Excessive amount of water

Недостатня кількість білків Not enough proteins

Недостатня кількість жирів Not enough fats

Надмірна кількість вуглеводів Excessive amount of carbohydrates

Недостатня кількість жирів і білків Insufficient amount of fats and proteins

49 / 200
До клініки поступив чоловік 40-ка років, якого укусила гадюка. Де переважно буде проходити гемоліз еритроцитів у цьому випадку? A 40-year-old man who was bitten by a viper came to the clinic. Where will hemolysis of erythrocytes mainly take place in this case?

У клітинах селезінки In spleen cells

У кровоносному руслі In the bloodstream

У клітинах печінки In liver cells

У кістковому мозку In the bone marrow

У паренхімі нирок In the kidney parenchyma

50 / 200
При клінічному обстеженні пацієнта 70-ти років виявлено порушення рухових функцій, що пов’язано з віковими змінами у гіаліновому хрящі. Які вікові зміни викликали обмеження рухів у суглобах? During the clinical examination of a 70-year-old patient, a violation of motor functions was detected, which is associated with age-related changes in hyaline cartilage. What age-related changes caused the limitation of joint movements?

Потовщення охрястя Thickening of plaque

Збільшення кількості хрящових клітин Increase in the number of cartilage cells

Відкладання солей кальцію в міжклітинній речовині Deposition of calcium salts in the intercellular substance

Збільшення кількості ізогенних груп Increase in the number of isogenic groups

Збільшення гідрофільності основної речовини Increasing hydrophilicity of the main substance

51 / 200
Хворому 42-х років для лікування бактеріальної пневмонії призначено амоксицилін. Вкажіть, який механізм бактерицидної дії препарату? A 42-year-old patient was prescribed amoxicillin for the treatment of bacterial pneumonia. Specify the mechanism of bactericidal action of the drug?

Пригнічення SH-груп ферментів мікроорганізмів Suppression of SH-groups of enzymes of microorganisms

Пригнічення внутрішньоклітинного синтезу білка Inhibition of intracellular protein synthesis

Пригнічення синтезу клітинної стінки мікроорганізмів Suppression of the synthesis of the cell wall of microorganisms

Порушення проникливості цитоплазматичної мембрани Cytoplasmic membrane permeability violation

Антагонізм із параамінобензойною кислотою Antagonism with para-aminobenzoic acid

52 / 200
У жінки обмежений кровотік у нирках, підвищений артеріальний тиск. Гіперсекреція якого гормону зумовила підвищення тиску? The woman has limited blood flow in the kidneys, high blood pressure. Hypersecretion of which hormone caused the increase in pressure?

Ренін Ренін

Адреналін Adrenaline

Вазопресин Vasopressin

Еритропоетин Erythropoietin

Норадреналін Noradrenaline

53 / 200
У чоловіка 41-го року відзначаються періодичні напади серцебиття (пароксизми), сильне потовиділення, напади головного болю. При обстеженні виявлена гіпертензія, гіперглікемія, підвищення основного обміну, тахікардія. При якій патології наднирників спостерігається подібна картина? A 41-year-old man has periodic attacks of heart palpitations (paroxysms), profuse sweating, and attacks of headache. During the examination, hypertension, hyperglycemia, increased basic metabolism, tachycardia were found In what pathology of the adrenal glands is a similar picture observed?

Гіперфункція кори наднирників Hyperfunction of the adrenal cortex

Гіперфункція мозкового шару Hyperfunction of the cerebral layer

Гіпофункція кори наднирників Hypofunction of the adrenal cortex

Первинний альдостеронізм Primary aldosteronism

Гіпофункція мозкового шару Hypofunction of the cerebral layer

54 / 200
Пацієнт 16-ти років, що страждає на хворобу Іценко-Кушінга, консультований з приводу надмірної ваги тіла. При опитуванні з’ясувалося, що енергетична цінність спожитої їжі складає 1700-1900 ккал/добу. Яка провідна причина ожиріння у даному випадку? A 16-year-old patient suffering from Itsenko-Cushing's disease is consulted about excess body weight. The survey revealed that the energy value of the consumed food is 1700-1900 kcal/day. What is the leading cause of obesity in this case?

Гіподинамія Hypodynamia

Надлишок глюкокортикоїдів Glucocorticoid excess

Нестача глюкокортикоїдів Lack of glucocorticoids

Надлишок інсуліну Excess insulin

Нестача інсуліну Lack of insulin

55 / 200
У людини з нападом бронхоспазму необхідно зменшити вплив блукаючого нерва на гладеньку мускулатуру бронхів. Які мембранні циторецептори доцільно заблокувати для цього? In a person with an attack of bronchospasm, it is necessary to reduce the influence of the vagus nerve on the smooth muscle of the bronchi. Which membrane cytoreceptors should be blocked for this purpose?

М-холінорецептори M-cholinergic receptors

Н-холінорецептори N-cholinoreceptors

β-адренорецептори β-adrenergic receptors

α-адренорецептори α-adrenergic receptors

α- та β-адренорецептори α- and β-adrenergic receptors

56 / 200
У новонародженої дитини виявлено наступну патологію: аномалія розвитку нижньої щелепи та гортані, що супроводжується характерними змінами голосу, а також мікроцефалія, вада серця, чотирьохпалість. Найбільш імовірною причиною таких аномалій є делеція: The following pathology was detected in a newborn child: an anomaly of the development of the lower jaw and larynx, accompanied by characteristic changes in the voice, as well as microcephaly, a heart defect, four fingers. The most likely cause of such the anomaly is a deletion:

Короткого плеча 5-ої хромосоми Short arm of the 5th chromosome

21-ої хромосоми 21st chromosome

Короткого плеча 7-ої хромосоми Short arm of the 7th chromosome

Короткого плеча 9-ої хромосоми Short arm of the 9th chromosome

Короткого плеча 11-ої хромосоми Short arm of the 11th chromosome

57 / 200
У жінки з важкою інтоксикацією, зумовленою сепсисом, який і послужив безпосередньою причиною смерті, на розтині виявлене 'тигрове серце'. Мікроскопічно в цитоплазмі кардіоміоцитів виявлені ліпіди. Який морфогенетичний механізм розвитку переважно лежить в основі даної дистрофії? In a woman with severe intoxication caused by sepsis, which was the direct cause of death, a 'tiger heart' was found at autopsy. Microscopically, lipids were found in the cytoplasm of cardiomyocytes. What morphogenetic the mechanism of development mainly underlies this dystrophy?

Трансформація Transformation

Декомпозиція Decomposition

Спотворений синтез Distorted synthesis

- -

Інфільтрація Infiltration

58 / 200
Чоловік 42-х років з ураженням ліктьового нерва не може звести до серединної лінії II і V пальці. Функція яких м’язів при цьому порушена? A 42-year-old man with damage to the ulnar nerve cannot bring the II and V fingers to the midline. Which muscles are impaired?

Короткий долонний м’яз Short Long M'az

Тильні міжкісткові м’язи Back interosseous muscles

Долонні міжкісткові м’язи Palmaric interosseous muscles

Червоподібні м’язи Vermiform muscles

Відвідний м’яз мізинця Abductor little finger

59 / 200
Хворому з гострою недостатністю надниркових залоз був призначений лікарський препарат, після застосування якого у нього з’явилися скарги на біль у кістках (двічі були переломи), часті простудні хвороби, набряки, повільне загоювання ран. Який препарат міг спричинити такі явища? A patient with acute adrenal insufficiency was prescribed a drug, after using which he complained of bone pain (twice had fractures), frequent colds , edema, slow wound healing. What drug could have caused such phenomena?

Ретаболіл Retabolil

Преднізолон Prednisone

Спіронолактон Spironolactone

Тестостерон Testosterone

Естріол Estriol

60 / 200
У подружжя народився син, хворий на гемофілію. Батьки здорові, а дідусь за материнською лінією також хворий на гемофілію. Визначте тип успадкування ознаки. The couple had a son with hemophilia. The parents are healthy, and the maternal grandfather also has hemophilia. Determine the type of inheritance of the trait.

Неповне домінування Incomplete dominance

Домінантний, зчеплений зі статтю Sex-linked dominant

Аутосомно-домінантний Autosomal dominant

Рецесивний, зчеплений зі статтю Sex-linked recessive

Аутосомно-рецесивний Autosomal recessive

61 / 200
Хворому з діагнозом цукровий діабет II типу ендокринолог призначив глібенкламід. Вкажіть основний механізм дії цього засобу: The endocrinologist prescribed glibenclamide to a patient diagnosed with type II diabetes. Specify the main mechanism of action of this agent:

Пригнічує глюконеогенез Suppresses gluconeogenesis

Активує транспорт глюкози в клітину Activates glucose transport into the cell

Підсилює захоплення глюкози периферичними тканинами Strengthens glucose uptake by peripheral tissues

Підсилює метаболізм глюкози Strengthens glucose metabolism

Стимулює секрецію інсуліну бета-клітинами острівців Лангерганса Stimulates insulin secretion by beta cells of the islets of Langerhans

62 / 200
У мазку з нальоту на мигдаликах хворого з підозрою на дифтерію виявлено палички синього кольору з потовщеннями на полюсах. Який метод фарбування мазків було використано? In a smear from a plaque on the tonsils of a patient suspected of diphtheria, blue rods with thickenings at the poles were found. What method of staining the smears was used?

Леффлера Lefflera

Гінса Гінса

Буррі Буррі

Нейссера Neisser

Грама Gram

63 / 200
Дитина народилася в стані асфіксії. Який препарат необхідно ввести новонародженому для стимуляції дихання? The child was born in a state of asphyxia. What drug should be administered to the newborn to stimulate breathing?

Празозин Prazosin

Атропін Atropine

Прозерин Prozerin

Лобелін Lobelin

Етимізол Etimizole

64 / 200
Хворий похилого віку страждає на хронічний закреп, в основі якого лежить гіпотонія товстого кишечнику. Який препарат слід призначити хворому? An elderly patient suffers from chronic constipation, the basis of which is hypotonia of the large intestine. What drug should be prescribed to the patient?

Ацеклідин Aceclidine

Натрію сульфат Sodium sulfate

Бісакодил Bisacodyl

Касторова олія Castor oil

Прозерин Prozerin

65 / 200
У пацієнта 38-ми років при прогресуванні стафілококового гнійного періодонтиту виникло гнійне запалення кістково-мозкових просторів альвеолярного відростка, а потім тіла нижньої щелепи. Мікроскопічно кісткові балки витончені, вогнища некрозу, кісткові секвестри, оточені сполучнотканинною капсулою. Який найбільш імовірний діагноз? A 38-year-old patient developed purulent inflammation of the bone-marrow spaces of the alveolar process and then the body of the lower jaw during the progression of staphylococcal purulent periodontitis. Microscopically, the bone beams are thin, foci necrosis, bone sequestrations surrounded by a connective tissue capsule. What is the most likely diagnosis?

Гнійний періостит Suppurative periostitis

Хронічний остеомієліт Chronic osteomyelitis

Пародонтома Periodontoma

Хронічний фіброзний періостит Chronic fibrous periostitis

Гострий остеомієліт Acute osteomyelitis

66 / 200
У немовляти виявлено мікроцефалію. Лікарі вважають, що це пов’язано з застосуванням жінкою під час вагітності актиноміцину Д. На які зародкові листки в першу чергу подіяв цей тератоген? The baby was diagnosed with microcephaly. Doctors believe that this is due to the woman's use of actinomycin D during pregnancy. Which germ layers were primarily affected by this teratogen?

Ектодерма Ектодерма

Мезодерма Mesoderm

Ентодерма та мезодерма Entoderm and Mesoderm

Ентодерма Entoderma

Усі листки All leaves

67 / 200
У чоловіка 65-ти років впродовж 15-ти років була виражена артеріальна гіпертензія. Останнім часом систолічний тиск почав знижуватися, а діастолічний залишився підвищеним. Який гемодинамічний тип артеріальної гіпертензії у хворого? A 65-year-old man had severe arterial hypertension for 15 years. Recently, systolic pressure began to decrease, and diastolic pressure remained elevated. What is the hemodynamic type of arterial hypertension in the patient?

Еукінетичний Eukinetic

- -

Гіперкінетичний Hyperkinetic

Гіпокінетичний Hypokinetic

Нормокінетичний Normokinetic

68 / 200
У жінки 52-х років при обстеженні було виявлено зниження кількості еритроцитів у крові та підвищення рівня вільного гемоглобіну в плазмі крові (гемоглобінемія). КП- 0,85. Який вид анемії спостерігається у хворої? In a 52-year-old woman, the examination revealed a decrease in the number of erythrocytes in the blood and an increase in the level of free hemoglobin in the blood plasma (hemoglobinemia). CP- 0.85. What type of anemia is observed in the patient?

Набута гемолітична Acquired hemolytic

Спадкова гемолітична Hereditary hemolytic

Гостра постгеморагічна Acute posthemorrhagic

Анемія внаслідок порушення ери-тропоезу Anemia due to violation of erythropoiesis

Хронічна постгеморагічна Chronic posthemorrhagic

69 / 200
У бактеріологічній лабораторії проводиться дослідження якості питної води. Її мікробне число виявилося близько 100. Які мікроорганізми враховувалися при цьому? The quality of drinking water is being tested in the bacteriological laboratory. Its microbial count was found to be about 100. What microorganisms were taken into account?

Бактерії групи кишкової палички Escherichia coli bacteria

Всі бактерії, що виросли на живильному середовищі All bacteria grown on nutrient medium

Ентеропатогенні бактерії та вїруси Enteropathogenic bacteria and viruses

Бактерії, патогенні для людей та тварин Bacteria pathogenic for humans and animals

Умовно-патогенні мікроорганізми Conditionally pathogenic microorganisms

70 / 200
В гістологічному препараті визначається орган, стінка якого утворена трьома оболонками. Внутрішня оболонка складається з ендотелію і тонкого підендотеліального шару. Зовнішня оболонка найтовстіша. Який орган представлений у препараті? In the histological preparation, an organ is identified, the wall of which is formed by three membranes. The inner membrane consists of endothelium and a thin subendothelial layer. The outer membrane is the thickest. What organ is represented in the preparation?

Сечовід Ureter

Артерія Artery

Матка Mother

Серце Heart

Вена Вена

71 / 200
Лімфоцит уражений ретровірусом ВІЛ (СНІД). В цьому випадку напрямок потоку інформації в клітині буде: The lymphocyte is affected by the retrovirus HIV (AIDS). In this case, the direction of the flow of information in the cell will be:

Поліпептид -> РНК -> ДНК -> і-РНК Polypeptide -> RNA -> DNA -> i-RNA

ДНК -> і-РНК -> поліпептид -> ДНК DNA -> i-RNA -> polypeptide -> DNA

ДНК -> поліпептид -> і-РНК DNA -> polypeptide -> i-RNA

і-РНК -> поліпептид -> ДНК i-RNA -> polypeptide -> DNA

РНК -> ДНК -> і-РНК -> поліпептид RNA -> DNA -> i-RNA -> polypeptide

72 / 200
Під час судово-медичної експертизи жінки, яка загинула у автокатастрофі, знайдено ембріон на стадії ранньої гаструли. Назвіть місце його локалізації при умові нормального розвитку: During the forensic medical examination of a woman who died in a car accident, an embryo at the stage of early gastrula was found. Name the place of its localization under the condition of normal development:

Яєчник Ovary

Черевна порожнина Abdominal cavity

Стінка матки Wall of uterus

Маткова частина яйцепроводу Uterine part of fallopian tube

Ампульна частина яйцепроводу Ampoule part of oviduct

73 / 200
В основі розвитку імунних і алергічних реакцій організмом застосовуються однакові механізми відповіді імунної системи на антиген. Визначте основну відмінність алергічних реакцій від імунних? In the basis of the development of immune and allergic reactions, the body uses the same mechanisms of the immune system's response to an antigen. Define the main difference between allergic reactions and immune reactions?

Особливість будови антигенів Particularity of the structure of antigens

Розвиток пошкодження тканин Development of tissue damage

Кількість антигену, що потрапляє Number of incoming antigen

Шляхи потрапляння в організм Paths of entering the body

Спадкова схильність Hereditary tendency

74 / 200
У пацієнта внаслідок черепно-мозкової травми знижена шкірна чутливість. Яка ділянка кори великого мозку може бути ураженою? The patient has decreased skin sensitivity as a result of a brain injury. Which part of the cerebral cortex can be affected?

Поясна звивина Land curvature

Лобна ділянка кори Frontal cortex

Передня центральна звивина Anterior central gyrus

Задня центральна звивина Posterior central gyrus

Потилична ділянка Occipital region

75 / 200
При розтині трупа чоловіка 47-ми років, померлого раптово, в інтимі черевного відділу аорти знайдені осередки жовтого кольору у вигляді плям та смуг, що не вибухають над поверхнею інтими. При фарбуванні суданом ІІІ спостерігається жовтогаряче забарвлення. Для якої стадії атеросклерозу характерні такі зміни? At the autopsy of a 47-year-old man who died suddenly, yellow cells were found in the intima of the abdominal part of the aorta in the form of spots and stripes that did not explode above the intima surface When stained with Sudan III, a yellow-red color is observed. What stage of atherosclerosis is characterized by such changes?

Атероматозу Atheromatosis

Ліпосклерозу Liposclerosis

Атерокальцинозу Atherocalcinosis

Ліпоїдозу Lipoidosis

Стадія утворення атероматозної виразки Stage of formation of atheromatous ulcer

76 / 200
При огляді ротової порожнини стоматолог виявив появу у дитини перших великих нижніх кутніх зубів. Який вік дитини? When examining the oral cavity, the dentist discovered the appearance of the child's first large lower angular teeth. What is the child's age?

4-5 років 4-5 years

6-7 років 6-7 years

12-13 років 12-13 years

8-9 років 8-9 years

10-11 років 10-11 years

77 / 200
У батьків, хворих на гемоглобінопатію (аутосомно-домінантний тип успадкування), народилася здорова дівчинка. Які генотипи батьків? Parents suffering from hemoglobinopathy (autosomal dominant type of inheritance) had a healthy girl. What are the parents' genotypes?

Батько гетерозиготний за геном гемоглобінопатії, у матері цей ген відсутній The father is heterozygous for the hemoglobinopathy gene, the mother does not have this gene

У обох батьків ген гемоглобінопатії відсутній Both parents have no hemoglobinopathy gene

Мати гетерозиготна за геном гемоглобінопатії, у батька цей ген відсутній The mother is heterozygous for the hemoglobinopathy gene, the father lacks this gene

Обоє гомозиготні за геном гемоглобінопатії Both are homozygous for the hemoglobinopathy gene

Обоє гетерозиготні за геном гемоглобінопатії Both are heterozygous for the hemoglobinopathy gene

78 / 200
Хворому з артеріальною гіпертензією було призначено один з антигіпертензивних засобів. Артеріальний тиск нормалізувався, однак хворого почав турбувати постійний сухий кашель. Який з перерахованих препаратів має таку по6ічну дію? A patient with arterial hypertension was prescribed one of the antihypertensive drugs. Blood pressure normalized, but the patient began to be bothered by a constant dry cough. Which of the listed drugs has such a side effect?

Лізиноприл Lisinopril

Ніфедипін Nifedipine

Резерпін Backup

Анаприлін Anaprilin

Клофелін Clofelin

79 / 200
У 30-річного померлого наркомана, який страждав на ВІЛ-інфекцію, при патоморфологічному дослідженні виявлено, що обидві легені ущільнені, темно-бордовосірі, мало повітряні, міжальвеолярні перетинки густо інфільтровані лімфоцитами, частина альвеолоцитів трансформовані у великі клітини з центрально розташованим круглим ядром зі світлим обідцем, які нагадують 'совине око'. Яка опортуністична інфекція викликала пневмонію у даного хворого? In a 30-year-old deceased drug addict who suffered from HIV infection, a pathomorphological examination revealed that both lungs were compacted, dark burgundy-gray, with little air, interalveolar membranes densely infiltrated with lymphocytes, part of the alveolocytes transformed into large cells with a centrally located round nucleus with a light rim, which resemble an 'owl's eye'. What opportunistic infection caused pneumonia in this patient?

Токсоплазма Toxoplasma

Цитомегаловірус Cytomegalovirus

Герпес-вірус Herpes virus

Атипова мікобактерія Atypical mycobacterium

Пневмоциста карінії Carinia pneumocyst

80 / 200
У пацієнта у результаті тривалого блювання відбувається значна втрата шлункового соку, що є причиною порушення кислотно-лужного стану в організмі. Яка з перерахованих форм порушення кислотно-лужного стану має місце? The patient has a significant loss of gastric juice as a result of prolonged vomiting, which is the cause of acid-base disturbance in the body. Which of the listed forms of acid-base disturbance has place?

Негазовий алкалоз Nongaseous alkalosis

Метаболічний ацидоз Metabolic acidosis

Негазовий ацидоз Nongaseous acidosis

Газовий алкалоз Gas alkalosis

Газовий ацидоз Gas acidosis

81 / 200
Хвора звернулася зі скаргами на болі у правій латеральній ділянці живота. Під час пальпації визначається щільне, нерухоме, пухлиноподібне утворення. У ділянці якого відділу травної трубки можлива наявність пухлини? The patient complained of pain in the right lateral part of the abdomen. During palpation, a dense, immobile, tumor-like formation is detected. In which part of the digestive tube is the presence of a tumor possible?'

Colon sigmoideum Colon sigmoideum

Caecum Blind

Colon descendens Descending colon

Colon transversum Transverse colon

Colon ascendens Colon ascending

82 / 200
На практичному занятті з мікробіології студентам запропоновано пофарбувати суміш бактерій за методикою Грама та пояснити механізм фарбування. Які морфологічні структури бактерій зумовлюють грамнегативне та грампозитивне фарбування бактерій? In a practical lesson in microbiology, students were asked to stain a mixture of bacteria using the Gram method and explain the staining mechanism. What morphological structures of bacteria cause gram-negative and gram-positive staining of bacteria?

Капсула Capsule

ЦПМ ЦПМ

Клітинна стінка Cell wall

Джгутики Flags

Цитоплазма Cytoplasm

83 / 200
Студентка 22-х років звернулася до лікаря із скаргами на підвищення температури тіла до 38oC, слабкість, біль у горлі. Об’єктивно: язик вкритий білим нальотом. Які гістологічні структури язика беруть участь в утворенні цього нальоту? A 22-year-old student turned to the doctor with complaints of an increase in body temperature to 38oC, weakness, sore throat. Objectively: the tongue is covered with a white coating. What are the histological structures of the tongue involved in the formation of this plaque?

Епітелій ниткоподібних сосочків Epithelium of filiform papillae

Епітелій грибоподібних сосочків Epithelium of mushroom-shaped papillae

Епітелій листоподібних сосочків Epithelium of leaf-shaped papillae

Епітелій жолобкуватих сосочків Epithelium of grooved papillae

Сполучнотканинна основа всіх сосочків язика Connective tissue base of all tongue papillae

84 / 200
Досліджуються рецептори, інформація від яких прямує до кори без участі таламусу. Які це рецептори? Receptors from which information is sent to the cortex without the involvement of the thalamus are being studied. What are these receptors?

Зорові Visual

Слухові Aural

Нюхові Olfactory

Смакові Tasty

Дотикові Touch

85 / 200
Препарат 'Гептрал', який використовують при хворобах печінки, містить Sаденозилметіонін. Ця активна амінокислота бере участь у синтезі: The drug 'Heptral', which is used for liver diseases, contains Sadenosylmethionine. This active amino acid is involved in the synthesis of:

Фосфоліпідів Phospholipids

Жовчних кислот Bile acids

Холестерину Cholesterol

Триацилгліцеролів Triacylglycerols

Гему Гему

86 / 200
Хвора 39-ти років, з цукровим діабетом в анамнезі, госпіталізована до клініки у прекоматозному стані кетоацидотичного типу. Збільшення вмісту якого метаболіту до цього призвело? A 39-year-old patient with a history of diabetes mellitus was admitted to the clinic in a precomatose state of the ketoacidotic type. The increase in the content of which metabolite led to this?

Ацетоацетат Ацетоацетат

Альфа-кетоглутарат Alpha-Ketoglutarate

Малонат Malone

Цитрат Citrate

Аспартат Aspartate

87 / 200
В хірургічне відділення доставлено пацієнта з рiзаною раною медіального краю передпліччя. При обстеженні виявлено, що в хворого перерізано ліктьовий м’яз-згинач зап’ястка і ліктьовий м’яз-розгинач зап’ястка. Які з рухів кисті будуть порушені у хворого? A patient with a cut wound of the medial edge of the forearm was brought to the surgical department. During the examination, it was found that the patient's wrist flexor ulnar muscle and ulnar muscle were cut' 'Which of the movements of the hand will be impaired in the patient?

Розгинання Expansion

Розгинання і відведення Extension and retraction

Згинання Bending

Відведення Deduction

Приведення Account

88 / 200
Для діагностування деяких хромосомних хвороб використовують визначення статевого хроматину. Назвіть хворобу, при якій потрібне це визначення: Sex chromatin determination is used to diagnose some chromosomal diseases. Name the disease that requires this determination:

Хвороба Дауна Down's disease

Гемофілія Hemophilia

Хвороба Брутона Bruton's disease

Синдром Шерешевського-Тернера Shereshevsky-Turner syndrome

Трисомія Е Trisomy E

89 / 200
У хворої після видалення матки розвинулась гостра анурія (немає виділення сечі). Які анатомічні структури найімовірніше було пошкоджено при операції? After hysterectomy, the patient developed acute anuria (no urine output). What anatomical structures were most likely damaged during the operation?

Сечоводи Ureters

Зовнішній сфінктер сечівника External urethral sphincter

Сечівник Urine

Цибулинно-губчастий м’яз Onion spongy muscle

Внутрішній сфінктер сечівника Internal urethral sphincter

90 / 200
Дихальний коефіцієнт у хворого складає 0,7. Це свідчить, що у клітинах людини переважає: The patient's respiratory rate is 0.7. This indicates that human cells are dominated by:

Змішане окислення жирів та білків Mixed oxidation of fats and proteins

Змішане окислення жирів та вуглеводів Mixed oxidation of fats and carbohydrates

Окислення білків Protein oxidation

Окислення жирів Oxidation of fats

Окислення вуглеводів Oxidation of carbohydrates

91 / 200
У чоловіка при обстеженні виявлено порушення кровообігу міокарда лівого передсердя. У басейні якої артерії відбулись порушення кровообігу? During the examination, a man was found to have a violation of the blood circulation of the left atrial myocardium. In the basin of which artery did the blood circulation violations occur?

Ліва вінцева Left coronal

- -

Передня міжшлуночкова гілка лівої вінцевої артерії Anterior interventricular branch of the left coronary artery

Права вінцева Coronal right

Права та ліва вінцеві Right and left corona

92 / 200
Лікар-цитогенетик при виготовленні метафазної пластинки обробив культуру лейкоцитів гіпотонічним (0,56%) розчином хлориду калію. Після цього відбулося набухання клітин і розрив клітинної мембрани за рахунок надходження води до клітини. Який механізм транспорту має місце в цьому випадку? During the production of the metaphase plate, the cytogeneticist treated the leukocyte culture with a hypotonic (0.56%) solution of potassium chloride. After that, the cells swelled and the cell membrane ruptured due to the influx of water to the cell. What transport mechanism takes place in this case?

Дифузія Diffusion

Полегшена дифузія Diffusion facilitated

Ендоосмос Ендоосмос

Фагоцитоз Phagocytosis

Піноцитоз Pinocytosis

93 / 200
При зовнішньому дослідженні трупа чоловіка 69-ти років, який помер 4 години тому, патологоанатом відмітив, що м’язи померлого мають дуже щільну консистенцію, суглоби згинаються та розгинаються важко. Як називається ця патологоанатомічна ознака смерті? During the external examination of the corpse of a 69-year-old man who died 4 hours ago, the pathologist noted that the muscles of the deceased have a very dense consistency, the joints bend and flex difficult. What is the name of this pathological sign of death?

Трупне розкладання Decomposition

Трупне висихання Corpse desiccation

Трупні гіпостази Corpse Hypostases

Трупне охолодження Corporal cooling

Трупне заклякання Corpse Charm

94 / 200
Швидкість проведення збудження нервовими волокнами становить 120 м/сек. Який з наведених чинників, перш за все, забезпечує таку швидкість? The speed of conduction of excitation by nerve fibers is 120 m/sec. Which of the following factors primarily ensures this speed?

Великий фактор надійності Large reliability factor

Малий поріг деполяризації Small depolarization threshold

Великий потенціал спокою Great rest potential

Наявність мієлінової оболонки Presence of myelin sheath

Велика амплітуда потенціалу дії Large action potential amplitude

95 / 200
У померлої дитини 3-х років за життя мала місце менінгіальна симптоматика, На розтині в м’якій мозковій оболонці макроскопічно виявлені просоподібні вузлики, які мікроскопічно представлені осередком казеозного некрозу з валами епітеліоїдних, лімфоїдних клітин, між якими зустрічаються великі клітини з ядрами на периферії у вигляді півмісяця. Який найбільш імовірний менінгіт у дитини? The deceased child had meningeal symptoms for 3 years during his life. At autopsy, millet-like nodules were macroscopically found in the soft meninges, which microscopically represented a caseous necrosis center with shafts of epithelioid, lymphoid cells, between which there are large cells with nuclei on the periphery in the form of a crescent. What is the most likely meningitis in a child?

Сифілітичний Syphilitic

Бруцельозний Brucellosis

Грипозний Gripoznia

Туберкульозний Tuberculosis

Менінгококовий Meningococcal

96 / 200
Лікар записав в історії хвороби, що у хворого дихання поверхневе (знижена глибина дихання). Це означає, що зменшеним є такий показник зовнішнього дихання: The doctor recorded in the medical history that the patient's breathing is shallow (reduced depth of breathing). This means that the external breathing rate is reduced:

Хвилинний об’єм дихання Minute respiratory volume

Функціональна залишкова ємність Functional residual capacity

Життєва ємність легень Vital lung capacity

Дихальний об’єм Respiratory volume

Ємність вдиху Inhalation capacity

97 / 200
При проведенні амніоцентезу в клітинах плоду виявлено по 2 тільця статевого хроматину (тільця Барра). Для якого захворювання характерна дана ознака? During amniocentesis, 2 bodies of sex chromatin (Barr bodies) were detected in fetal cells. For which disease is this sign characteristic?

Синдром Клайнфельтера Klinefelter Syndrome

Синдром Дауна Down Syndrome

Синдром Шерешевського-Тернера Shereshevsky-Turner syndrome

Синдром Патау Patau Syndrome

Трисомія X Trisomy X

98 / 200
Відомо, що типові патологічні процеси розвиваються за однаковими закономірностями в різних органах і тканинах та у різних видів тварин. Яке з перерахованих явищ можна віднести до типового патологічного процесу? It is known that typical pathological processes develop according to the same patterns in different organs and tissues and in different species of animals. Which of the listed phenomena can be attributed to a typical pathological process?

Пухлина Tumor

Інфаркт міокарда Myocardial infarction

Непрохідність кишківника Intestinal obstruction

Туберкульоз Tuberculosis

Гіпертонічна хвороба Hypertensive disease

99 / 200
У хворого 49-ти років на гострий панкреатит виникала загроза некрозу підшлункової залози, що супроводжувалось надходженням у кров і тканини активних панкреатичних протеїназ і розщеплення тканинних білків. Які захисні фактори організму можуть інгібувати ці процеси? A 49-year-old patient with acute pancreatitis was at risk of pancreatic necrosis, which was accompanied by the entry into the blood and tissues of active pancreatic proteinases and the breakdown of tissue proteins. What are the protective factors body can inhibit these processes?

Кріоглобулін, інтерферон Cryoglobulin, interferon

Імуноглобуліни Immunoglobulins

Гемоплексин, гаптоглобін Hemoplexin, haptoglobin

Церулоплазмін, трансферин Ceruloplasmin, transferrin

α2-макроглобулін, α1-антитрипсин α2-macroglobulin, α1-antitrypsin

100 / 200
У 19-місячної дитини із затримкою розвитку та проявами самоагресії, вміст сечової кислоти в крові -1,96 ммоль/л. При якому метаболічному порушенні це спостерігається? In a 19-month-old child with developmental delay and manifestations of self-aggression, the content of uric acid in the blood is -1.96 mmol/l. In which metabolic disorder is this observed?

Подагра Gout

Синдром набутого імунодефіциту Acquired immunodeficiency syndrome

Синдром Леша-Ніхана Lesch-Nyhan syndrome

Хвороба Іценко-Кушінга Itsenko-Cushing disease

Хвороба Гірке Bitter disease

101 / 200
Чоловік 53-х років звернувся зі скаргами на гострий біль у правому підребер’ї. При огляді лікар звернув увагу на пожовтілі склери хворого. Лабораторні аналізи показали підвищену активність АЛТ та негативну реакцію на стеркобілін у калі. Для якого захворювання характерні такі симптоми? A 53-year-old man complained of acute pain in the right hypochondrium. During the examination, the doctor drew attention to the patient's yellowed sclera. Laboratory tests showed increased activity of ALT and a negative reaction to stercobilin in feces. What disease is characterized by such symptoms?

Хронічний коліт Chronic Colitis

Гепатит Hepatitis

Хронічний гастрит Chronic gastritis

Гемолітична жовтяниця Hemolytic Jaundice

Жовчнокам’яна хвороба Cholelithiasis

102 / 200
Основними тригерами, що включають ефекторні системи клітини у відповідь на дію гормонів, є протеїнкінази, які змінюють каталітичну активність певних регуляторних ферментів шляхом АТФ-залежного фосфорилювання. Який із наведених ферментів є активним у фосфорильованій формі? The main triggers involving the cell's effector systems in response to hormones are protein kinases, which change the catalytic activity of certain regulatory enzymes through ATP-dependent phosphorylation. Which of the following of enzymes is active in phosphorylated form?

Глікогенфосфорилаза Glycogen phosphorylase

Піруваткіназа Pyruvate Kinase

ГОМГ-КоА-редуктаза GOMG-CoA-reductase

Ацетил-КоА-карбоксилаза Acetyl-CoA-carboxylase

Глікогенсинтаза Glycogen synthase

103 / 200
У пацієнта встановлено порушення синтезу та виділення вазопресину. В якому відділі нефрона найбільше порушиться процес сечоутворення? The patient has been diagnosed with a violation of the synthesis and release of vasopressin. In which part of the nephron will the process of urine formation be most disturbed?

Клубочок Ball

Тонка частина петлі Генле Thin part of the loop of Henle

Проксимальний звивистий каналець Proximal convoluted tubule

Збірна трубочка Collection tube

Товста частина петлі Генле Thick part of the loop of Henle

104 / 200
При обстеженні 2-х місячної дитини педіатр звернула увагу, що плач дитини нагадує котячий крик. Діагностовані мікроцефалія і вада серця. За допомогою цитогенетичного метода з’ясований каріотип дитини 46, XX, 5р-. Дане захворювання є наслідком такого процесу: During the examination of a 2-month-old child, the pediatrician noticed that the child's cry resembled a cat's cry. Microcephaly and a heart defect were diagnosed. The child's karyotype was determined using the cytogenetic method 46, XX, 5p-. This disease is a consequence of the following process:

Інверсія Inversion

Плейотропія Pleiotropy

Делеція Deletion

Транслокація Translocation

Дуплікація Duplication

105 / 200
В експерименті подразнюють гілочки симпатичного нерва, які іннервують серце. Це призвело до збільшення сили серцевих скорочень, тому що через мембрану типових кардіоміоцитів збільшився: In the experiment, the branches of the sympathetic nerve, which innervate the heart, are irritated. This led to an increase in the force of heart contractions, because through the membrane of typical cardiomyocytes increased:

Вихід іонів калію Output of potassium ions

Вихід іонів кальцію Output of calcium ions

Вхід іонів кальцію та калію Input of calcium and potassium ions

Вхід іонів калію Input of potassium ions

Вхід іонів кальцію Input of calcium ions

106 / 200
У пацієнта 60-ти років виявлено збільшення порогу сприймання звуків високої частоти. Зміна функцій яких структур слухового аналізатора зумовлює виникнення цього порушення? In a 60-year-old patient, an increase in the threshold for the perception of high-frequency sounds was detected. A change in the functions of which auditory analyzer structures causes this disturbance?

Барабанної перетинки Eardrum

Органу Корті ближче до гелікотре-ми The organ of Corti is closer to the helicotrema

Органу Корті ближче до овального віконця The organ of Corti is closer to the oval window

М’язів середнього вуха Middle ear muscles

Євстахієвої труби Eustachian tube

107 / 200
У пацієнта виявлено кишкову непрохідність, знижений апетит, нудоту, блювання. На основі проведеної лабораторної діагностики встановлено дифілоботріоз. Зараження відбулось через вживання: Intestinal obstruction, reduced appetite, nausea, vomiting were detected in the patient. Based on the laboratory diagnosis, diphyllobotriosis was established. The infection occurred through consumption of:

Крабів та раків Crabs and crayfish

Свинини Pork

Яловичини Beef

Яєць Egg

Риби Риби

108 / 200
В інфекційну клініку доставлено хворого із проявами лихоманки, що повторюється вдруге з інтервалом 2 дні. В краплі крові, зафарбованій за Романовським-Гімзою, виявлено звивисті клітини синьо-фіолетового кольору. Який мікроорганізм викликав захворювання? A patient was brought to the infectious disease clinic with manifestations of fever, which recurred for the second time with an interval of 2 days. In a drop of blood stained according to Romanovsky-Giemsa, tortuous cells of blue-violet color. What microorganism caused the disease?

Treponema pallidum Treponema paleum

Borrelia recurentis Recurring Borrelia

Plasmodium vivax Plasmodium vivax

Leptospira interrogans Questioning Leptospira

Ricketsia typhi Ricketsia typhi

109 / 200
У юнака 18-ти років діагностовано хворобу Марфана. При дослідженні встановлено: порушення розвитку сполучної тканини, будови кришталика ока, аномалії серцево- судинної системи, арахнодактилія. Яке генетичне явище зумовило розвиток цієї хвороби? An 18-year-old young man was diagnosed with Marfan's disease. During the examination, it was found: a violation of the development of connective tissue, the structure of the lens of the eye, anomalies of the cardiovascular system, arachnodactyly. What genetic phenomenon caused the development of this disease?

Плейотропія Pleiotropy

Кодомінування Codominance

Комплементарність Complementarity

Неповне домінування Incomplete dominance

Множинний алелізм Multiple allelism

110 / 200
При алкаптонурії відбувається надмірне виділення гомогентизинової кислоти із сечею. З порушенням метаболізму якої амінокислоти пов’язано виникнення цього захворювання? With alkaptonuria, there is an excessive excretion of homogentisic acid in the urine. The occurrence of this disease is associated with a violation of the metabolism of which amino acid?

Тирозин Tyrosine

Метіонін Methionine

Фенілаланін Phenylalanine

Аланін Alanine

Аспарагін Aparagin

111 / 200
Чоловік 30-ти років, водій за професією, страждає на алергічний риніт із загостренням у весняний період. Лікар призначив хворому антигістамінний засіб з незначним седативним ефектом та тривалістю дії близько 24 годин. Який із перерахованих засобів було призначено? A 30-year-old man, a driver by profession, suffers from allergic rhinitis with exacerbation in the spring period. The doctor prescribed the patient an antihistamine with a slight sedative effect and a duration of action of about 24 hours. Which of the listed means was prescribed?

Окситоцин Oxytocin

Лоратадин Loratadine

Гепарин Heparin

Димедрол Diphenhydramine

Вікасол Вікасол

112 / 200
Хворий госпіталізований з небезпекою поширення запального процесу з потиличної ділянки в порожнину черепа. Крізь яке анатомiчне утворення можливе це поширення? The patient is hospitalized with the risk of spreading the inflammatory process from the occipital region to the skull cavity. Through which anatomical formation is this spread possible?

Круглий отвір Round hole

Виростковий канал Condylar canal

Овальний отвір Oval hole

Остистий отвір Prickly Hole

Тім ’яний отвір That hole

113 / 200
Після введення пеніцилину в хворого розвився набряк Квінке. Який препарат екстреної терапії необхідно ввести хворому? After the administration of penicillin, the patient developed Quincke's edema. What emergency therapy drug should be administered to the patient?

Сульфацил-натрій Sulfacyl sodium

Преднізолон Prednisone

Рифампіцин Rifampicin

Аскорбінова кислота Ascorbic acid

Но-шпа No-shpa

114 / 200
Під час електронномікроскопічного дослідження біоптату гепатоцитів на біліарному полюсі виявлено велику кількість плоских цистерн, сплющених у центральній частині й розширених на периферії, та дрібних міхурців із секреторними гранулами. Назвіть цю структуру: During an electron microscopic examination of a biopsy of hepatocytes at the biliary pole, a large number of flat cisternae, flattened in the central part and expanded at the periphery, and small vesicles with secretory granules were found. Name this structure:

Лізосома Lysosome

Піноцитозні міхурці Pinocytotic vesicles

Ендоплазматична сітка Endoplasmic reticulum

Мікротрубочки Microtubules

Комплекс Гольджі Golgi Complex

115 / 200
У кішки з децеребраційною ригідністю потрібно знизити тонус м’язів. Цього можна досягти шляхом: A cat with decerebrate rigidity needs to reduce muscle tone. This can be achieved by:

Подразнення вестибулярних ядер Дейтерса Irritation of vestibular nuclei of Deiters

Подразнення отолітових вестибулорецепторів Irritation of otolith vestibuloreceptors

Подразнення ампулярних вестибулорецепторів Irritation of ampullary vestibuloreceptors

Подразнення вестибулослухового нерва Vestibulo-auditory nerve irritation

Руйнування вестибулярних ядер Дейтерса Destruction of vestibular nuclei of Deiters

116 / 200
При гастробіопсії у хворого встановлена метаплазія поверхневого епітелію слизової оболонки, який замість циліндричного набув вигляду кишкового. Разом з тим спостерігається склероз на місці залоз слизової оболонки та лімфогістіоцитарна інфільтрація. Про яке захворювання шлунка можна думати? During a gastrobiopsy, the patient was diagnosed with metaplasia of the surface epithelium of the mucous membrane, which, instead of being cylindrical, took on the appearance of an intestinal one. At the same time, there was sclerosis at the site of the glands of the mucous membrane and lymphohistiocytic infiltration. About what kind of stomach disease can you think of?

Ерозивний гастрит Erosive Gastritis

Поверхневий хронічний гастрит Superficial chronic gastritis

Хронічний атрофічний гастрит Chronic atrophic gastritis

Хронічний гастрит з ураженням залоз без атрофії Chronic gastritis with gland damage without atrophy

Корозивний гастрит Corrosive Gastritis

117 / 200
При дослідженні імунного статусу людини обов’язково визначають кількість імуноглобулінів різних класів. Яка з перерахованих реакцій використовується для цього? When examining a person's immune status, the number of immunoglobulins of different classes must be determined. Which of the listed reactions is used for this?

Ланцюгово-полімеразна Polymerase chain

Подвійної імунодифузії Double immunodiffusion

Бласттрансформації Blast transformations

Оберненої непрямої гемаглютинації Reverse indirect hemagglutination

Радіальної імунодифузії Radial immunodiffusion

118 / 200
У хворого, що страждає на спадкову хворобою Хартнупа, спостерігаються пелагроподібні ураження шкіри, порушення розумового розвитку в результаті нестачі нікотинової кислоти. Причиною цього захворювання є порушення такого процесу: A patient suffering from hereditary Hartnup's disease has pellagra-like lesions on the skin, impaired mental development as a result of a lack of nicotinic acid. The cause of this disease is a violation of the following process:

Всмоктування і реабсорбція цистеїну Absorption and reabsorption of cysteine

Трансамінування фенілаланіну Phenylalanine transamination

Всмоктування і реабсорбція в нирках триптофану Absorption and reabsorption in the kidneys of tryptophan

Декарбоксилювання триптофану Decarboxylation of tryptophan

Всмоктування і реабсорбція в нирках метіоніну Absorption and reabsorption in kidneys of methionine

119 / 200
Хворий помилково прийняв надмірну дозу тироксину. До яких змін секреції тиреоліберину та тиреотропіну це призведе? The patient mistakenly took an excessive dose of thyroxine. What changes in the secretion of thyroliberin and thyrotropin will this lead to?

Секреція тиреотропіну збільшиться, тиреоліберину - зменшиться Secretion of thyrotropin will increase, thyroliberin will decrease

Секреція тиреоліберину збільшиться, тиреотропіну - зменшиться Secretion of thyroliberin will increase, thyrotropin will decrease

Змін секреції гормонів не буде There will be no changes in hormone secretion

Секреція гормонів зменшиться Hormone secretion will decrease

Секреція гормонів збільшиться Hormone secretion will increase

120 / 200
Проводять дуоденальне зондування. Що із наведеного доцільно ввести людині під шкіру, щоб суттєво збільшити надходження до дванадцятипалої кишки жовчі? Duodenal sounding is being performed. Which of the following should be injected under the skin of a person to significantly increase the flow of bile to the duodenum?

Нейротензин Neurotensin

Холецистокінін-панкреозимін Cholecystokinin-pancreozymin

Гастрин Gastrin

Соматостатин Somatostatin

Секретин Secretin

121 / 200
У чоловіка 32-х років, хворого на пневмонію, спостерігається закупорка харкотинням дихальних шляхів. В організмі хворого при цьому буде розвиватися така зміна кислотнолужної рівноваги: A 32-year-old man with pneumonia has a blockage of the respiratory tract with sputum. In this case, the patient's body will develop the following change in the acid-base balance:

Респіраторний алкалоз Respiratory alkalosis

Респіраторний ацидоз Respiratory acidosis

Метаболічний алкалоз Metabolic alkalosis

Метаболічний ацидоз Metabolic acidosis

Змін не буде There will be no changes

122 / 200
При аналізі ЕКГ виявлено випадіння деяких серцевих циклів PQRST. Наявні зубці і комплекси не змінені. Назвіть вид аритмії: During the ECG analysis, the loss of some PQRST cardiac cycles was detected. The existing teeth and complexes are not changed. Name the type of arrhythmia:

Синоатріальна блокада Sinoatrial block

Миготлива аритмія Atrial fibrillation

Передсердна екстрасистола Atrial extrasystole

Внутрішньопередсердна блокада Intra-atrial blockade

Атріовентрикулярна блокада Atrioventricular block

123 / 200
У хворої встановлено порушення виділення тиреотропного гормону гіпофіза. Зі зниженням функцій якої частки гіпофіза це пов’язано? The patient has been diagnosed with a violation of the release of thyroid-stimulating hormone from the pituitary gland. This is due to a decrease in the functions of which part of the pituitary gland?

Infundibulum Funnel

Pars intermedia Pars intermedia

- -

Lobus posterior Lobus posterior

Lobus anterior Lobus anterior

124 / 200
У пацієнтки 23-х років після використання нової губної помади з’явилися набряк і свербіння губ, а через 2 дні - кірочки на червоній облямівці губ. Який тип алергічної реакції найбільш імовірний? A 23-year-old patient developed swelling and itching of the lips after using a new lipstick, and after 2 days - crusts on the red border of the lips. What type of allergic reactions most likely?

Стимулюючий Stimulating

Сповільнений Delayed

Цитотоксичний Cytotoxic

Імунокомплексний Immunocomplex

Анафілактичний Anaphylactic

125 / 200
У хворого лікар виявив накопичення рідини в плевральній порожнині справа над куполом діафрагми. У якому анатомічному утворі накопичилась рідина? In the patient, the doctor discovered fluid accumulation in the pleural cavity on the right above the dome of the diaphragm. In which anatomical structure did the fluid accumulate?

Реброво-середостінний синус Cost-mediastinal sinus

Діафрагмально-середостінний синус Diaphragmatic-mediastinal sinus

Реброво-діафрагмальний лівий синус Cost-phrenic left sinus

Реброво-діафрагмальний правий синус Cost-phrenic right sinus

- -

126 / 200
При регенерації епітелію слизової оболонки порожнини рота (розмноження клітин) відбулася реплікація (авторепродукція) ДНК за напівконсервативним механізмом. При цьому нуклеотиди нової нитки ДНК є комплементарними до: During the regeneration of the epithelium of the mucous membrane of the oral cavity (cell reproduction), replication (autoreproduction) of DNA took place by a semi-conservative mechanism. At the same time, the nucleotides of the new DNA strand are complementary to:

Змістовних кодонів Content codons

Ферменту РНК-полімерази RNA polymerase enzyme

Материнської нитки Mother threads

Інтронних ділянок гену Intron regions of the gene

Ферменту ДНК-полімерази DNA polymerase enzyme

127 / 200
В результаті травми порушено цілісність переднього корінця спинного мозку. Які відростки яких нейронів при цьому пошкоджені? As a result of the injury, the integrity of the anterior root of the spinal cord is violated. Which processes of which neurons are damaged?

Дендрити чутливих нейронів Dendrites of sensory neurons

Дендрити вставних нейронів Dendrites of intercalated neurons

Аксони чутливих нейронів Axons of sensory neurons

Дендрити рухових нейронів Dendrites of motor neurons

Аксони рухових нейронів Axons of motor neurons

128 / 200
При визначенні повітряної та кісткової провідності звуку було встановлено, що у пацієнта ліве вухо краще сприймає звук при кістковому його проведенні, що могло бути пов’язане з захворюванням: When determining the air and bone conduction of sound, it was established that the patient's left ear perceives sound better with bone conduction, which could be related to the disease:

Зовнішнього вуха справа Right external ear

Середнього вуха справа Right middle ear

Внутрішнього вуха справа Right inner ear

Середнього вуха зліва Left middle ear

Внутрішнього вуха зліва Left inner ear

129 / 200
Чоловік 55-ти років доставлений до реанімаційного відділення без свідомості. Зі слів родичів стало відомо, що хворий помилково випив метиловий спирт. Який антидот необхідно використати в даному випадку? A 55-year-old man was brought to the intensive care unit unconscious. From the words of relatives, it became known that the patient mistakenly drank methyl alcohol. What antidote should be used in this case?'

Етанол Етанол

Ацетилцистеїн Acetylcysteine

Протаміну сульфат Protamine Sulfate

Налоксон Naloxone

Тетурам Тетурам

130 / 200
Хвора на ревматоїдний артрит після трьохтижневого лікування преднізолоном почала скаржитись на перебої в ро6оті серця. З чим пов’язаний розвиток даного небажаного ефекту препарату? After a three-week treatment with prednisone, a patient with rheumatoid arthritis began to complain of heart rhythm disturbances. What is the cause of the development of this undesirable effect of the drug?

Гіперурікемія Hyperuricemia

Гіпоглікемія Hypoglycemia

Гіперкаліємія Hyperkalemia

Гіпокаліємія Hypokalemia

Гіперглікемія Hyperglycemia

131 / 200
Для профілактики атеросклерозу, ішемічної хвороби серця, порушень мозкового кровообігу рекомендується споживання жирів із високим вмістом поліненасичених жирних кислот. Однією з таких жирних кислот є: For the prevention of atherosclerosis, coronary heart disease, disorders of cerebral circulation, it is recommended to consume fats with a high content of polyunsaturated fatty acids. One of these fatty acids is:

Лауринова Laurinova

Стеаринова Stearin

Пальмітоолеїнова Palmitooleinova

Лінолева Linoleum

Олеїнова Oleinova

132 / 200
Обстежуваний знаходиться у фазі швидкохвильового сну. При цьому на ЕЕГ реєструється: The subject is in the phase of REM sleep. At the same time, the following is recorded on the EEG:

δ-хвиля δ-wave

θ-хвиля θ-hour

β-хвиля β-wave

α-веретено α-spindle

α-хвиля α-wave

133 / 200
Внаслідок ДТП у потерпілої 37-ми років виникло неутримання сечі. Які сегменти спинного мозку пошкоджені? As a result of a road accident, a 37-year-old victim developed urinary incontinence. What segments of the spinal cord are damaged?

L1 — L2 L1 — L2

Th2 — Th5 Th2 — Th5

S2 - S4 S2 - S4

Th1 — Th5 Th1 — Th5

Th1 — L1 Th1 — L1

134 / 200
Жінці 26-ти років, хворій на бронхіт, призначили засіб етіотропної терапії - антибіотик широкого спектру дії. Який це препарат? A 26-year-old woman with bronchitis was prescribed a means of etiotropic therapy - a broad-spectrum antibiotic. What is this drug?

Інтерферон Interferon

Дексаметазон Dexamethasone

Доксициклін Doxycycline

Амброксол Ambroxol

БЦЖ-вакцина BCG vaccine

135 / 200
При обстеженні жінки 56-ти років, що хвора на цукровий діабет 1-го типу, виявлене порушення білкового обміну, що при лабораторному дослідженні крові проявляється аміноацидемією а клінічно - уповільненням загоєння ран і зменшенням синтезу антитіл. Який з перерахованих механізмів викликає розвиток аміноацидемії? During the examination of a 56-year-old woman with type 1 diabetes, a violation of protein metabolism was detected, which was manifested by aminoacidemia during a laboratory blood test, and clinically - slowing down wound healing and reducing the synthesis of antibodies. Which of the listed mechanisms causes the development of aminoacidemia?

Збільшення ліпопротеїдів високої щільності High-density lipoprotein increase

Зменшення концентрації амінокислот у крові Decreased concentration of amino acids in blood

Підвищення протеолізу Proteolysis increase

Підвищення онкотичного тиску в плазмі крові Increased oncotic pressure in blood plasma

Гіперпротеїнемія Hyperproteinemia

136 / 200
Хворому на туберкульоз легень було призначено препарат з групи антибіотиків, що відноситься до високоефективних протитуберкульозних засобів. Який препарат був призначений? A drug from the group of antibiotics, which belongs to highly effective anti-tuberculosis drugs, was prescribed to a patient with pulmonary tuberculosis. What drug was prescribed?

Цефалексин Cephalexin

ПАСК ПАСК

Рифампіцин Rifampicin

Циклосерин Cycloserine

Тетрациклін Tetracycline

137 / 200
При дослідженні тимуса дитини 5-ти років, що померла від гострої деструктивної стафілококової пневмонії, виявлено зменшення маси залози до 3,0 г. При гістологічному дослідженні в тимусі знайдено: зменшення часточок залози, значне зменшення кількості лімфоцитів, з колапсом строми часточок, інверсія шарів, кистоподібне збільшення тілець Гассаля. Який з перелічених діагнозів найбільш імовірний? When examining the thymus of a 5-year-old child who died of acute destructive staphylococcal pneumonia, a decrease in the mass of the gland to 3.0 g was found. During histological examination in the thymus, : reduction of gland lobules, a significant decrease in the number of lymphocytes, with collapse of the stroma of the lobules, inversion of layers, cyst-like enlargement of Hassal's corpuscles. Which of the listed diagnoses is the most probable?

Тимомегалія Thymomegaly

Акцидентальна інволюція тимусу Accidental thymus involution

Гіпоплазія тимусу Thymus hypoplasia

Дисплазія тимусу Thymic dysplasia

Агенезія тимусу Agenesis of the thymus

138 / 200
При обстеженні хворого окуліст виявив збільшення часу адаптації ока до темряви. Нестача якого вітаміну може бути причиною такого симптому? During the examination of the patient, the ophthalmologist found an increase in the eye's adaptation time to the dark. A lack of which vitamin can be the cause of such a symptom?

A A

K K

C C

B6 B6

B2 B2

139 / 200
Хворому 63-х років з атонією сечового мiхура лікар призначив препарат, дозу якого хворий самостійно збільшив. З’явились підвищене потовиділення, салівація, діарея, м’язові спазми. Препарат якої групи був призначений? A 63-year-old patient with atony of the urinary bladder was prescribed a drug, the dose of which the patient increased on his own. Increased sweating, salivation, diarrhea, muscle spasms appeared What group of drugs was prescribed?

Адреноблокатори Adrenoblockers

Реактиватори холінестерази Cholesterase Reactivators

Гангліоблокатори Ganglioblockers

Холіноміметики Cholinomimetics

Токолітики Tocolytics

140 / 200
На мікропрепараті плівки пухкої сполучної тканини видно клітину овальної форми, зі світлою цитоплазмою і великим ядром що містить специфічний малюнок гетерохроматину у вигляді циферблату годинника (або спиць колеса). Яка клітина у полі зору? A micrograph of a film of loose connective tissue shows an oval-shaped cell with light cytoplasm and a large nucleus containing a specific pattern of heterochromatin in the form of a clock face (or spokes of a wheel). What a cell in the field of view?

Макрофаг Macrophage

Адипоцит Adipocyte

Фібробласт Fibroblast

Тканинний базофіл Tissue Basophil

Плазмоцит Plasmocyte

141 / 200
Для лікування піодермії лікар призначив вакцину, яка виготовлена зі штаму бактерій, виділених від хворого. До якого типу вакцин належить даний препарат? To treat pyoderma, the doctor prescribed a vaccine made from a strain of bacteria isolated from the patient. What type of vaccine does this drug belong to?

Атенуйована вакцина Vaccine Attenuated

Хімічна вакцина Chemical vaccine

Генноінженерна вакцина Genetically engineered vaccine

Аутовакцина Autovaccine

Асоційована вакцина Associated vaccine

142 / 200
У хворого скарги на загальну слабкість, підвищену втому, зниження апетиту і маси тіла. В анамнезі часті пневмонії. На підставі клінічних даних та результатів дослідження периферійної крові у нього діагностовано хронічний лімфолейкоз. Які дегенеративні зміни лейкоцитів характерні для даного захворювання? The patient complains of general weakness, increased fatigue, decreased appetite and body weight. He has a history of frequent pneumonia. On the basis of clinical data and the results of a peripheral blood test, he was diagnosed with chronic lymphocytic leukemia. What degenerative changes in leukocytes are characteristic of this disease?

Тіні Боткіна-Гумпрехта Shadows of Botkin-Humprecht

Токсична зернистість Toxic granularity

Палички Ауера Auer rods

Зерна Амато Grains of Amato

Тільця Князькові-Деле Tiltsia Knyazkovi-Dele

143 / 200
У чоловіка 72-х років довготривала хронічна патологія легень призвела до недостатності клапанів легеневої артерії і трикуспідального клапану, недостатності кровообігу за правошлуночковим типом. Який тип артеріальної гіпертензії є причиною перевантаження серця об’ємом? In a 72-year-old man, long-term chronic lung pathology led to insufficiency of the valves of the pulmonary artery and tricuspid valve, insufficiency of blood circulation according to the right ventricular type. What type of arterial hypertension is the cause of overload the volume of the heart?

Есенціальна гіпертензія Essential hypertension

Центрально-ішемічна гіпертензія Central ischemic hypertension

Сольова гіпертензія Saline hypertension

Рефлексогенна гіпертензія Reflexogenic hypertension

Легенева гіпертензія Pulmonary hypertension

144 / 200
Хворому для лікування серцевої недостатності було призначено серцевий глікозид. Яка супутня патологія може сприяти кумуляції серцевих глікозидів? The patient was prescribed a cardiac glycoside for the treatment of heart failure. What concomitant pathology can contribute to the accumulation of cardiac glycosides?

Гіперацидний гастрит Hyperacid Gastritis

Гіпоацидний гастрит Hypoacid gastritis

Ниркова недостатність Kidney failure

Анорексія Anorexia

Гіпертонічна хвороба Hypertensive disease

145 / 200
Чоловік 40-ка років хворіє на гіперацидний гастрит з нічними голодними болями. Призначте хворому лікарський засіб - блокатор гістамінових Н2 -рецепторів III покоління, який знизить виділення хлористоводневої кислоти (особливо вночі) та збільшить утворення захисного слизу: A 40-year-old man suffers from hyperacid gastritis with nocturnal hunger pangs. Prescribe the patient a medicine - a blocker of histamine H2 receptors of the III generation, which will reduce the release of hydrochloric acid ( especially at night) and will increase the formation of protective mucus:

Пірензепін Pirenzepine

Фамотидин Фамотидин

Платифіліну гідротартрат Platiphylline hydrotartrate

Атропіну сульфат Atropine sulfate

Метацин Methacin

146 / 200
П’ятирічна дитина-правша після черепно-мозкової травми на деякий час втратила здатність розмовляти, але через тривалий час ця здатність у неї відновилась. Яка півкуля була травмована й за рахунок якої властивості ЦНС дітей відновлення мови стало можливим? A five-year-old right-handed child lost the ability to speak for a while after a brain injury, but after a long time this ability was restored. Which hemisphere was injured and due to which properties of the children's central nervous system, the restoration of speech became possible?

Ліва півкуля, інертність Left hemisphere, inertia

Права півкуля, рухливість Right hemisphere, mobility

Ліва півкуля, пластичність Left hemisphere, plasticity

Права півкуля, пластичність Right hemisphere, plasticity

Обидві півкулі, інертність Both hemispheres, inertia

147 / 200
Чоловік 35-ти років помер у приймальному відділенні лікарні, куди був доставлений у шоковому стані. На розтині тіла виявлено аневризму дуги аорти з її розривом та масивною кровотечею. Мікроскопічно: в медії аорти скупчення лімфоцитів та плазмоцитів, поодинокі гігантські клітини Пирогова-Лангханса, фібробласти. Спостерігаються також дрібні вогнища некрозу та руйнування еластичних структур стінки аорти. Яке захворювання зумовило такі зміни? A 35-year-old man died in the hospital's reception department, where he was brought in a state of shock. An autopsy revealed an aneurysm of the aortic arch with its rupture and massive bleeding. Microscopically : in the media of the aorta, accumulations of lymphocytes and plasma cells, single giant cells of Pirogov-Langhans, fibroblasts. There are also small foci of necrosis and destruction of the elastic structures of the aortic wall. What disease caused such changes?

Атеросклероз Atherosclerosis

- -

Туберкульоз Tuberculosis

Висипний тиф Whispering Typhus

Сифіліс Syphilis

148 / 200
У дитини 5-ти років розвинулось гостре респіраторне захворювання, яке супроводжувалось кашлем, виділенням значної кількості слизу із носа. Який тип запалення у хворої дитини? A 5-year-old child developed an acute respiratory disease, which was accompanied by a cough, discharge of a significant amount of mucus from the nose. What type of inflammation does the sick child have?

Катаральне Cataral

Гнилісне Rotten

Геморагічне Hemorrhagic

Фібриноїдне Fibrinoid

Гнійне Suppurative

149 / 200
Чоловікові 58-ми років зроблено операцію з приводу раку простати. Через 3 місяці йому проведено курс променевої та хіміотерапії. До комплексу лікарських препаратів входив 5- фтордезоксиуридин - інгібітор тимідилатсинтази. Синтез якої речовини блокується цим препаратом? A 58-year-old man underwent an operation for prostate cancer. After 3 months, he underwent a course of radiation and chemotherapy. The complex of drugs included 5-fluorodeoxyuridine - a thymidylate synthase inhibitor . The synthesis of which substance is blocked by this drug?

ДНК ДНК

- -

р-РНК р-РНК

т-РНК т-РНА

і-РНК i-RNA

150 / 200
Стеатоз виникає внаслідок накопичення триацилгліцеролів у гепа-тоцитах. Одним з механізмів розвитку цього захворювання є зменшення утилізації нейтрального жиру ЛПДНЩ. Які ліпотропні речовини попереджують розвиток стеатозу? Steatosis occurs as a result of the accumulation of triacylglycerols in hepatocytes. One of the mechanisms of the development of this disease is a decrease in the utilization of neutral VLDL fat. What lipotropic substances prevent the development of steatosis?

Метіонін, B6 , B12 Methionine, B6, B12

Валін, B3 , B2 Валін, B3 , B2

Аргінін, B2 , B3 Arginin, B2 , B3

Аланін, B1 , PP Аланін, B1 , PP

Ізолейцин, B1 , B2 Isoleucine, B1 , B2

151 / 200
У пацієнта, що прибув з ендемічного за малярією району, підвищилася температура тіла, відзначається головний біль, озноб, загальне нездужання - симптоми, що характерні й для звичайної застуди. Які лабораторні дослідження необхідно провести, щоб підтвердити або спростувати діагноз 'малярія'? The patient, who arrived from a malaria-endemic area, has an elevated body temperature, headache, chills, general malaise - symptoms that are characteristic of a common cold. What laboratory tests should be conducted to confirm or deny the diagnosis of 'malaria'?

Аналіз сечі Urine analysis

Дослідження спинномозкової рідини Research of cerebrospinal fluid

Дослідження пунктату лімфовузлів Study of punctate lymph nodes

Мікроскопія мазків крові Microscopy of blood smears

Мікроскопія пунктату червоного кісткового мозку Red bone marrow punctate microscopy

152 / 200
У чоловіка 40-ка років внаслідок посиленого гемолізу еритроцитів підвищився вміст заліза в плазмі крові. Який білок забезпечує його депонування в тканинах? A 40-year-old man has increased iron content in blood plasma as a result of increased hemolysis of erythrocytes. What protein ensures its deposition in tissues?

Транскортин Transcortin

Трансферин Transferrin

Феритин Феритин

Гаптоглобін Haptoglobin

Альбумін Albumin

153 / 200
У дитячому дошкільному закладі напередодні новорічних свят було зареєстровано спалах кишкової інфекції. При бактеріологічному дослідженні випорожнень хворих патогенних бактерій не було виділено. При електронній мікроскопії виявлено утворення округлої форми з чітким обідком і товстою втулкою, які нагадують колесо. Вкажіть найбільш імовірний збудник даної інфекції: An outbreak of an intestinal infection was registered in a children's preschool on the eve of New Year's holidays. Bacteriological examination of the patient's feces did not reveal pathogenic bacteria. Electron microscopy revealed a rounded formation with a clear a rim and a thick hub that resemble a wheel. Specify the most likely causative agent of this infection:

Pvulgaris Pvulgaris

Coxsacki-virus Coxsacki-virus

E.coli E.coli

Rotavirus Rotavirus

Adenovirus Adenovirus

154 / 200
У біоптаті щитоподібної залози виявлено атрофію паренхіматозних елементів, дифузну інфільтрацію тканини залози лімфоцитами та плазматичними клітинами з утворенням в ній лімфоїдних фолікулів. Для якого захворювання є характерними наведені ознаки? Atrophy of parenchymal elements, diffuse infiltration of the gland tissue by lymphocytes and plasma cells with the formation of lymphoid follicles were found in the biopsy of the thyroid gland. For which disease are these symptoms characteristic?

Тиреоїдит Хасімото Hashimoto's thyroiditis

Хвороба Базедова Bazedov's disease

Тиреоїдит Ріделя Riedel's thyroiditis

Ендемічний зоб Endemic goiter

Аденома щитоподібної залози Thyroid adenoma

155 / 200
При гістологічному дослідженні біоптатів, взятих з потовщених країв виразки шлунка, виявлені невеликі гніздові скупчення різко атипових гіперхромних невеликих епітеліальних клітин, які розташовані серед дуже розвиненої строми. Визначте пухлину: On histological examination of biopsies taken from the thickened edges of a gastric ulcer, small nested clusters of sharply atypical hyperchromic small epithelial cells were found, which are located among a highly developed stroma. Identify the tumor:

Аденокарцинома Adenocarcinoma

Медулярний рак Medular Cancer

Недиференційована саркома Undifferentiated sarcoma

Скіррозний недиференційований рак Scarcerous undifferentiated cancer

Аденома Adenoma

156 / 200
При розтині тіла померлого у інфекційному відділенні виявлено: фібринозногнійний перитоніт; в слизовій оболонці клубової кишки багаточислені виразкові дефекти овальної форми до 3-5 см, які розташовані вздовж кишки і повторюють форму пейєрової бляшки, краї виразок рівні, заокруглені, дно чисте, представлене м’язовою або серозною оболонкою. В дні двох виразок знайдені отвори до 0,3 см в діаметрі. Для якого захворювання найбільш характерні ці зміни? At the autopsy of the body of the deceased in the infectious department, the following were found: fibrino-purulent peritonitis; in the mucous membrane of the ileum, numerous ulcerative defects of an oval shape up to 3-5 cm, which are located along the intestine and repeat the shape of a Peyer's plaque, the edges of the ulcers are smooth, rounded, the bottom is clean, represented by a muscular or serous membrane. Holes up to 0.3 cm in diameter were found at the bottom of two ulcers. For which disease are these changes most characteristic?

Хвороба Крона Crohn's disease

Дизентерія Dysentery

Неспецифічний виразковий коліт Nonspecific ulcerative colitis

Черевний тиф Typhoid

Паратиф Paratyphos

157 / 200
Хворий 78-ми років з хворобою Паркінсона приймає препарати левадопи (наком). Яка антипаркінсонічна дія у цього засобу? A 78-year-old patient with Parkinson's disease takes levadopa drugs (nakom). What is the antiparkinsonian effect of this drug?

Адреноблокуюча Adrenoblocker

М-холіноблокуюча M-anticholinergic

Допамінергічна Dopaminergic

Блокуюча гістамінергічна дія Blocking histaminergic action

М-холіноміметична M-cholinomimetic

158 / 200
У хворого 47-ми років виникла кишкова коліка на фоні гіпертонічної хвороби. Засоби якої з перерахованих груп найбільш доцільно використати для її купірування у даній ситуації? A 47-year-old patient developed intestinal colic on the background of hypertension. Which of the listed groups of drugs is the most appropriate to use to stop it in this situation?

Міотропні спазмолітики Myotropic antispasmodics

Адреноміметики Adrenomimetics

Антихолінестеразні засоби Anticholinesterase drugs

М-холиноміметики M-cholinomimetics

Симпатоміметики Sympathomimetics

159 / 200
Під час ректороманоскопії хворого зі скаргами на діарею виявлено, що слизова оболонка прямої і сигмоподібної кишок різко гіперемована, набрякла, вкрита великою кількістю слизу, а у деяких ділянках вкрита плівчастими накладаннями зеленуватого кольору. Про яке захворювання можна думати? During the rectoromanoscopy of a patient with complaints of diarrhea, it was found that the mucous membrane of the rectum and sigmoid intestine is sharply hyperemic, swollen, covered with a large amount of mucus, and in some areas covered with membranous greenish overlays. What disease can you think of?

Амебіаз Amebiasis

Черевний тиф Typhoid

Сальмонельоз Salmonellosis

Дизентерія Dysentery

Холера Cholera

160 / 200
У хворого на ЕКГ виявлено збільшення тривалості комплексу QRS. Наслідком чого це може бути? An increase in the duration of the QRS complex was detected in the patient's ECG. What could be the consequence?

Збільшення часу охоплення збудженням шлуночків Increasing the time of ventricular excitation

Збільшення збудливості передсердь Increased atrial excitability

Збільшення часу охоплення збудженням передсердь Increasing the time of atrial excitation

Збільшення збудливості шлуночків та передсердь Increased excitability of ventricles and atria

Порушення провідності у атріовентрикулярному вузлі Disruption of conduction in the atrioventricular node

161 / 200
Хвороба Андерсена належить до групи спадкових хвороб, що розвиваються внаслідок уродженої недостатності синтезу певних ферментів глікогенолізу. Недостатність якого ферменту є молекулярною основою цього глікогенозу? Andersen's disease belongs to a group of hereditary diseases that develop as a result of congenital insufficiency of the synthesis of certain enzymes of glycogenolysis. The insufficiency of which enzyme is the molecular basis of this glycogenosis?

Глюкозо-6-фосфатази Glucose-6-phosphatases

Аміло(1,4-1,6)трансглікозидаза Amyl(1,4-1,6)transglycosidase

Фосфофруктокіназа Phosphofructokinase

Лізосомальні глікозидази Lysosomal glycosidases

Глікогенсинтаза Glycogen synthase

162 / 200
У хворого з варикозним розширенням вен під час огляду нижніх кінцівок відзначається: ціаноз, пастозність, зниження температури шкіри, поодинокі петехії. Який розлад гемодинаміки має місце у хворого? In a patient with varicose veins, during the examination of the lower extremities, the following is noted: cyanosis, pastiness, a decrease in skin temperature, isolated petechiae. What hemodynamic disorder does the patient have?

Венозна гіперемія Venous hyperemia

Обтураційна ішемія Obstructive ischemia

Компресійна ішемія Compression ischemia

Артеріальна гіперемія Arterial hyperemia

Тромбоемболія Thromboembolism

163 / 200
Після прийому сульфаніламідів у хворого виникли лихоманка, блювання і стул з кров’ю. У крові: лейк.-0,9 • 109 /л (гранул.- 0,7 • 109 /л), лейкоаглютиніни. Який з термінів найбільш точно характеризує виявлені зміни у крові? After taking sulfonamides, the patient developed fever, vomiting, and stools with blood. In the blood: leuk.-0.9 • 109 /l (granules- 0.7 • 109 /l), leukoagglutinins. Which of the terms most accurately characterizes the detected changes in the blood?

Лейкоз Leukemia

- -

Агранулоцитоз Agranulocytosis

Лейкопенія Leukopenia

Гемодилюція Hemodilution

164 / 200
При гістологічному дослідженні органів і тканин померлої від ниркової недостатності молодої жінки, у якої прижиттєво виявлявся високий титр антинуклеарних антитіл, виявлені поширені фібриноїдні зміни в стінках судин. Відмічається ядерна патологія з вакуолізацією ядер, каріоре-ксисом, утворенням гематоксилінових тілець. Який найбільш імовірний діагноз? During a histological examination of the organs and tissues of a young woman who died of kidney failure, in whom a high titer of antinuclear antibodies was detected during life, widespread fibrinoid changes in the vessel walls were detected. Nuclear pathology is noted with vacuolization of nuclei, karyorrhexis, formation of hematoxylin bodies. What is the most likely diagnosis?

Облітеруючий ендартеріїт Endarteritis obliterans

Атеросклероз Atherosclerosis

Гіпертонічна хвороба Hypertensive disease

Вузликовий періартеріїт Nodular periarteritis

Системний червоний вовчак Systemic lupus erythematosus

165 / 200
У недоношеного немовляти спостерігається жовтяниця. З нестачею у нього якого ферменту це пов’язано? A premature baby is jaundiced. What enzyme is it lacking?

Каталаза Catalase

Лужна фосфатаза Alkaline phosphatase

НАД+- дегідрогеназа NAD+- dehydrogenase

Кисла фосфатаза Acid phosphatase

УДФ-трансглюкуронідаза UDF-transglucuronidase

166 / 200
У хворого пухлина грудного відділу стравоходу. Куди можуть безпосередньо метастазувати пухлинні клітини? The patient has a tumor of the thoracic esophagus. Where can tumor cells directly metastasize?

Nodi mediastinales Mediastinal nodes

Nodi intercostales Intercostal nodes

Nodi gastrici Gastric nodes

Nodi hepatici Hepatic nodes

Ductus thoracicus Ductus thoracicus

167 / 200
При дослідженні вмісту дванадцятипалої кишки людини знайдені найпростіші грушоподібної форми з парними ядрами, чотирма парами джгутиків. Між ядрами - дві опірні нитки, з вентрального боку розташований присмоктувальний диск. Який представник найпростіших виявлений у хворого? When studying the contents of the human duodenum, the simplest pear-shaped ones with paired nuclei, four pairs of flagella were found. Between the nuclei there are two resistance threads, on the ventral side there is a suction disk. What A representative of the protozoa was found in the patient?

Трипаносома Trypanosoma

Трихомонада кишкова Intestinal trichomonas

Токсоплазма Toxoplasma

Лейшманія Leishmania

Лямблія Lamblia

168 / 200
36-ти років має місце гіповітаміноз B2 168. . Причиною виникнення специфічних симптомів (ураження епітелію, слизових, шкіри, рогівки ока) імовірно є дефіцит: B2 168 hypovitaminosis has been occurring for 36 years. . The cause of specific symptoms (epithelial, mucous membrane, skin, cornea damage) is probably a deficiency:

Цитохромоксидази Cytochrome oxidases

Флавінових коферментів Flavin coenzymes

Цитохрому А1 Cytochrome A1

Цитохрому В Cytochrome B

Цитохрому С Cytochrome C

169 / 200
При диспансерному обстеженні у хворого знайдено цукор в сечі. Який найбільш імовірний механізм виявлених змін, якщо вміст цукру в крові нормальний? During the dispensary examination, sugar was found in the patient's urine. What is the most probable mechanism of the detected changes, if the blood sugar content is normal?

Порушення фільтрації глюкози в клубочковому відділі нефрона Disturbance of glucose filtration in the glomerular part of the nephron

Недостатня продукція інсуліну підшлунковою залозою Insufficient production of insulin by the pancreas

Інсулінорезистентність рецепторів клітин Insulin resistance of cell receptors

Гіперпродукція глюкокортикоїдів наднирниками Hyperproduction of glucocorticoids by the adrenal glands

Порушення реабсорбції глюкози в канальцях нефрона Disturbance of glucose reabsorption in nephron tubules

170 / 200
На розтині в серці виявлено наступні зміни: великий осередок некрозу білого кольору з червоною облямівкою, який захоплює всю товщу серцевого м’яза. На зовнішній оболонці серця - ознаки фібринозного перикардиту. Який найбільш імовірний діагноз? At autopsy, the following changes were found in the heart: a large center of white necrosis with a red border, which occupies the entire thickness of the heart muscle. On the outer shell of the heart, there are signs of fibrinous pericarditis. What is the most likely diagnosis?

Субепікардіальний інфаркт міокарда Subepicardial myocardial infarction

Міокардит Myocarditis

Субендокардіальний інфаркт міокарда Subendocardial myocardial infarction

Інтрамуральний інфаркт міокарда Intramural myocardial infarction

Трансмуральний інфаркт міокарда Transmural myocardial infarction

171 / 200
На електронній мікрофотографії представлена клітина нейрального походження. Термінальна частина дендрита клітини має циліндричну форму і складається з 1000 замкнутих мембранних дисків. Яка клітина зображена на мікрофотографії? The electron micrograph shows a cell of neural origin. The terminal part of the dendrite of the cell has a cylindrical shape and consists of 1000 closed membrane discs. Which cell is shown in the micrograph?

Нейрон передніх рогів спинного мозку Neuron of the anterior horns of the spinal cord

Паличкова нейросенсорна Rodular neurosensory

Нейрон спинномозкового вузла Spinal node neuron

Нейрон кори великих півкуль Neuron of the cortex of the large hemispheres

Колбочкова нейросенсорна Cone's Neurosensory

172 / 200
Порушення процесів мієлінізації нервових волокон призводить до неврологічних розладів і розумової відсталості. Такі симптоми характерні для спадкових і набутих порушень обміну: Disturbance of the processes of myelination of nerve fibers leads to neurological disorders and mental retardation. Such symptoms are characteristic of hereditary and acquired metabolic disorders:

Фосфатидної кислоти Phosphatic acid

Сфінголіпідів Sphingolipids

Нейтральних жирів Neutral fats

Вищих жирних кислот Higher fatty acids

Холестерину Cholesterol

173 / 200
У хворого 15-ти років концентрація глюкози натще 4,8 ммоль/л, через годину після цукрового навантаження - 9,0 ммоль/л, через 2 години - 7,0 ммоль/л, через 3 години - 4,8 ммоль/л. Ці показники характерні для такого захворювання: A 15-year-old patient has a fasting glucose concentration of 4.8 mmol/l, an hour after a sugar load - 9.0 mmol/l, after 2 hours - 7.0 mmol/l, after 3 hours - 4.8 mmol/l. These indicators are typical for such a disease:

Цукровий діабет I типу Type I diabetes

Прихований цукровий діабет Hidden diabetes

- -

Хвороба Іценко-Кушінга Itsenko-Cushing disease

Цукровий діабет II типу Type II diabetes

174 / 200
У чоловіка 48-ми років виявлено порушення периферичного кровообігу з обмеженням припливу артеріальної крові, при цьому має місце збліднення даної ділянки, зниження місцевої температури. Це порушення називається: A 48-year-old man was found to have a violation of peripheral blood circulation with a restriction of arterial blood flow, at the same time there is a pallor of this area, a decrease in local temperature. This violation is called:

Реперфузійний синдром Reperfusion syndrome

Венозна гіперемія Venous hyperemia

Стаз Status

Ішемія Ischemia

Сладж Сладж

175 / 200
Хворому хірург видалив порожнинне утворення печінки діаметром 2 см. Встановлено, що стінка порожнини утворена щільною волокнистою сполучною тканиною, вміст являє собою каламутну, густу, жовтувато-зеленуватого кольору рідину з неприємним запахом, яка мікроскопічно складається переважно з поліморфноядерних лейкоцитів. Якому патологічному процесу відповідають такі морфологічні зміни? The surgeon removed a 2-cm-diameter liver cavity from the patient. It was established that the wall of the cavity is formed by dense fibrous connective tissue, the contents are a cloudy, thick, yellowish-greenish liquid with an unpleasant smell, which microscopically consists mainly of polymorphonuclear leukocytes. What pathological process do such morphological changes correspond to?

- -

Флегмона Phlegmon

Хронічний абсцес Chronic abscess

Емпієма Empyema

Гострий абсцес Acute abscess

176 / 200
Лікар призначив пацієнту з хронічним бронхітом відхаркувальний засіб, який діє шляхом розщеплення дисульфідних зв’язків глікозаміногліканів харкотиння, зменшуючи цим його в’язкість, проте попередив хворого про можливий бронхоспазм при його використанні. Який засіб був призначений? The doctor prescribed an expectorant to a patient with chronic bronchitis, which works by splitting the disulfide bonds of glycosaminoglycans of sputum, thereby reducing its viscosity, but warned the patient about possible bronchospasm when using it. What remedy was prescribed?

Бромгексин Bromhexine

Ацетилцистеїн Acetylcysteine

Натрію гідрокарбонат Sodium bicarbonate

Лібексин Libexin

Трава термопсису Thermopsis Grass

177 / 200
Лікар призначив хворому з гострою серцевою недостатністю не-глікозидний кардіотонічний засіб, який безпосередньо стимулює β1 -адренорецептори міокарда, що збільшує кровообіг, діурез. Застосовується лише внутрішньовенно крапельно внаслідок швидкої інактивації в організмі. Який препарат призначив лікар? The doctor prescribed a non-glycoside cardiotonic drug to a patient with acute heart failure, which directly stimulates the β1-adrenoceptors of the myocardium, which increases blood circulation, diuresis. It is used only intravenously as a result of rapid inactivation in the body. What drug did the doctor prescribe?

Анаприлін Anaprilin

Адреналін Adrenaline

Корглікон Corglycon

Дигоксин Digoxin

Добутамін Dobutamine

178 / 200
Чоловік 40-ка років перебував у пульмонологічному відділенні з приводу рецидивуючої правосторонньої пневмонії. Помер від легенево-серцевої недостатності. На розтині в правій лєгєні визначається ділянка круглої форми 3х4 см. Вона являє собою порожнину з нерівними шорсткими краями, заповнену каламутною вершкоподібною жовто-зеленою рідиною. Мікроскопічно: стінка порожнини утворена тканиною легені з дифузною інфільтрацією лейкоцитами. Визначте патологічний процес у легені: A 40-year-old man was in the pulmonology department due to recurrent right-sided pneumonia. He died of pulmonary and heart failure. An autopsy revealed a 3x4 cm round area in the right lung . It is a cavity with uneven rough edges, filled with a cloudy creamy yellow-green liquid. Microscopically: the wall of the cavity is formed by lung tissue with diffuse infiltration of leukocytes. Define the pathological process in the lung:

Емпієма Empyema

Гангрена Gangrene

Інфаркт Heart attack

Хронічний абсцес Chronic abscess

Гострий абсцес Acute abscess

179 / 200
Робочий комунальної служби спустився в каналізаційний колодязь без засобів захисту і через деякий час знепритомнів. Лікарями швидкої допомоги діагностовано отруєння сірководнем. Який вид гіпоксії при цьому розвинувся? A utility service worker went down into a sewage well without protective equipment and after some time passed out. Emergency doctors diagnosed hydrogen sulfide poisoning. What type of hypoxia developed?

Респіраторний Respiratory

Перевантажувальний Overloading

Циркуляторний Circular

Гемічний Chemical

Тканинний Fabric

180 / 200
Під час розтину тіла жінки 52-х років, яка тривалий час хворіла на жовчно-кам’яну хворобу, було знайдено: макроскопічно - печінка помірно збільшена, деформована, поверхня органу горбиста, тканина щільна, на розрізі тканина коричнева з зеленим відтінком, складається з множинних вузликів діаметром 8-10 мм. Мікроскопічно - гепатоцелюлярні вузлики оточені прошарками сполучної тканини, яка містить збільшену кількість дрібних жовчних протоків з холестазом. Діагностуйте захворювання печінки: During the autopsy of the body of a 52-year-old woman who suffered from gallstone disease for a long time, it was found: macroscopically - the liver is moderately enlarged, deformed, the surface of the organ is bumpy, the tissue is dense, on cross-section the tissue is brown with a green tint, consists of multiple nodules with a diameter of 8-10 mm. Microscopically, the hepatocellular nodules are surrounded by layers of connective tissue, which contains an increased number of small bile ducts with cholestasis. Diagnose liver disease:

Постнекротичний цироз печінки Post-necrotic liver cirrhosis

Холелітіаз Cholelithiasis

Портальний цироз печінки Portal cirrhosis

Біліарний цироз печінки Biliary cirrhosis

Токсична дистрофія печінки Toxic liver dystrophy

181 / 200
При гістологічному дослідженні біоптату, отриманого із нижньої третини стравоходу 57- річного чоловіка із симптомами тривалого рефлюксу шлункового вмісту, виявлено наступні зміни: у слизовій оболонці на місці багатошарового плоского епітелію визначається одношаровий залозистий призматичний епітелій, з ознаками продукції слизу. Вкажіть патологічний процес, який виник у слизовій оболонці: During the histological examination of a biopsy obtained from the lower third of the esophagus of a 57-year-old man with symptoms of long-term reflux of gastric contents, the following changes were found: in the mucous membrane in place of the multi-layered squamous epithelium a single-layer glandular prismatic epithelium is determined, with signs of mucus production. Specify the pathological process that occurred in the mucous membrane:

Метаплазія Metaplasia

Гіперплазія Hyperplasia

Регенерація Regeneration

Гіпертрофія Hypertrophy

Організація Organization

182 / 200
Хворій жінці із захворюванням нирок, що супроводжується вираженими набряками, призначили діуретичний препарат, що пригнічує реабсорбцію в нирках іонів Na+ і води, посилює виведення нирками іонів K+ і Mg++, викликає гіперурикемію, зумовлює потужний діуретичний ефект. Назвіть цей препарат: A sick woman with kidney disease, accompanied by severe edema, was prescribed a diuretic drug that inhibits the reabsorption of Na+ ions and water in the kidneys, increases the excretion of K+ and Mg++ ions by the kidneys, causes hyperuricemia, causes a powerful diuretic effect. Name this drug:

Фуросемід Furosemide

Аллопуринол Allopurinol

Діакарб Diakarb

Спіронолактон Spironolactone

Тріамтерен Triamterene

183 / 200
Лікар-стоматолог для лікування гінгівіту призначив пацієнту препарат з протипротозойною та антибактеріальною діями, який може викликати відразу до алкоголю. Вкажіть препарат, який призначив лікар: To treat gingivitis, the dentist prescribed the patient a drug with antiprotozoal and antibacterial effects, which can cause an aversion to alcohol. Specify the drug prescribed by the doctor:

Лінкоміцину гідрохлорид Lincomycin hydrochloride

Левоміцетин Levomycetin

Цефтріаксон Ceftriaxone

Метронідазол Metronidazole

Тетрациклін Tetracycline

184 / 200
Аналіз ЕКГ хворого виявив відсутність зубця Р. Тривалість та амплітуда QRS комплексу та зубця Т відповідають нормі. Що є водієм ритму серця даного пацієнта? Analysis of the patient's ECG revealed the absence of the P wave. The duration and amplitude of the QRS complex and the T wave correspond to the norm. What is the driver of this patient's heart rhythm?

Волокна Пуркін’є Purkinje fibers

Синусовий вузол Sine Node

Передсердно-шлуночковий вузол Atrioventricular node

Міокард шлуночків Myocardium of ventricles

Пучок Гіса His Bundle

185 / 200
харчування призвів до зменшення іонів Ca2+ 185. в крові. До збільшення секреції якого гормону це призведе? nutrition led to a decrease in Ca2+ 185. ions in the blood. Which hormone will this lead to an increase in secretion?

Вазопресин Vasopressin

Паратгормон Parathyroid hormone

Тироксин Thyroxine

Тирокальцитонін Thyrocalcitonin

Соматотропін Somatotropin

186 / 200
Хворій 43-х років для лікування бронхопневмонії призначена бензилпеніциліну натрієва сіль. Який з вказаних побічних ефектів найбільш характерний для даного засобу? A 43-year-old patient was prescribed benzylpenicillin sodium salt for the treatment of bronchopneumonia. Which of the listed side effects is most characteristic of this drug?

Ураження печінки Liver damage

Агранулоцитоз Agranulocytosis

Неврит слухового нерва Acoustic Neuritis

Алергічні реакції Allergic reactions

Анемія Anemia

187 / 200
Пацієнт із захворюванням першого верхнього різця зліва скаржиться на сильні болі шкіри в ділянці надбрівної дуги з того ж боку. Реалізація якого виду рефлексів спричиняє вказані реакції? A patient with a disease of the first upper incisor on the left complains of severe pain in the skin in the area of the brow arch on the same side. Implementation of which type of reflexes causes the indicated reactions?

Сомато-вісцеральні Somato-visceral

Вісцеро-вісцеральні Viscero-visceral

Вісцеро-дермальні Viscero-dermal

Вісцеро-соматичні Viscero-somatic

Пропріоцептивні Proprioceptive

188 / 200
Потерпілий 19-ти років доставлений до травматологічного відділення з різаною раною трапецієподібного м’яза. Яка з фасцій шиї формує піхву для даного м’яза? A 19-year-old victim was brought to the trauma department with a cut wound of the trapezius muscle. Which of the neck fascia forms the sheath for this muscle?

Поверхнева пластинка шийної фасції Surface plate of cervical fascia

М’ язова частина передтрахейної пластинки шийної фасції Muscular part of the pretracheal plate of the cervical fascia

Передхребтова пластинка шийної фасції Prevertebral plate of cervical fascia

Вісцеральна частина передтрахейної пластинки шийної фасції Visceral part of the pretracheal plate of the cervical fascia

Сонна піхва шийної фасції Carotid sheath of cervical fascia

189 / 200
У хворого діагностовано ГРВІ. У сироватці крові знайдено імуноглобуліни класу М. Який період інфекційного процесу в даному випадку? The patient was diagnosed with SARS. Immunoglobulins of class M were found in the blood serum. What is the period of the infectious process in this case?

Продромальний Prodromal

Гострий Acute

Реконвалесценція Convalescence

Інкубаційний Incubation

Мікробоносійство Microcarriage

190 / 200
У хворого 34-х років після перенесеної кишкової інфекції, викликаної сальмонелами, стали згасати симптоми захворювання. Імуноглобуліни якого класу будуть виявлені в крові хворого в період реконвалесценції? In a 34-year-old patient, after an intestinal infection caused by salmonella, the symptoms of the disease began to disappear. What class of immunoglobulins will be detected in the patient's blood during the period of convalescence?

IgM IgM

IgG IgG

IgA IgA

IgD IgD

IgE IgE

191 / 200
До шпиталю було доставлено юнака 16-ти років, хворого на інсулінозалежний цукровий діабет. Рівень глюкози у крові пацієнта складав 18 ммоль/л. Хворому було введено інсулін. Дві години потому рівень глюкози зменшився до 8,2 ммоль/л, тому що інсулін: A 16-year-old boy with insulin-dependent diabetes was brought to the hospital. The patient's blood glucose level was 18 mmol/l. The patient was given insulin. Two hours later, the glucose level decreased to 8.2 mmol/l, because insulin:

Стимулює перетворення глюкози в печінціу глікоген та ТАГ Stimulates the conversion of glucose in the liver to glycogen and TAG

Стимулює розщеплення глікогену в печінці Stimulates the breakdown of glycogen in the liver

Стимулює транспорт глюкози через плазматичні мембрани в головному мозку та печінці Stimulates glucose transport across plasma membranes in the brain and liver

Гальмує синтез кетонових тіл із глюкози Inhibits the synthesis of ketone bodies from glucose

Стимулює розщеплення глікогену у м’язах Stimulates glycogen breakdown in muscles

192 / 200
Хворий на трансмуральний інфаркт міокарда лівого шлуночка переведений до відділення реанімації у важкому стані. АТ- 70/50 мм рт.ст., ЧСС- 56/хв., ЧД- 32/хв. Зазначте головну ланку в патогенезі кардіоген-ного шоку: A patient with a transmural myocardial infarction of the left ventricle was transferred to the intensive care unit in critical condition. Blood pressure - 70/50 mm Hg, heart rate - 56/min. ChD - 32/min. Specify the main link in the pathogenesis of cardiogenic shock:

Втрата води Water loss

Втрата електролітів Electrolyte loss

Крововтрата Blood loss

Падіння периферичного судинного опору Falling of peripheral vascular resistance

Падіння серцевого викиду Decreasing cardiac output

193 / 200
Обстеження пацієнта з високим артеріальним тиском показало в нього вторинну артеріальну гіпертензію. Причиною такого стану є ренін-продукуюча пухлина нирки. Що є головною ланкою в патогенезі вторинної артеріальної гіпертензії в хворого? An examination of a patient with high blood pressure showed that he has secondary arterial hypertension. The cause of this condition is a renin-producing kidney tumor. What is the main link in the pathogenesis of secondary arterial hypertension in the patient?

Гіпєрпродукція інсуліну Hyperproduction of insulin

Гіперпродукція кортизолу Hyperproduction of cortisol

Недостатня продукція катехоламінів Insufficient production of catecholamines

Недостатня продукція вазопресину Insufficient production of vasopressin

Гіперпродукція ангіотензину 2, альдостерону Hyperproduction of angiotensin 2, aldosterone

194 / 200
У чоловіка 29-ти років з ножовим пораненням шиї визначається кровотеча. При первинній обробці рани встановлено, що пошкоджена судина, розташована вздовж латерального краю груднинно-ключично-соскоподібного м’яза. Визначте цю судину: Bleeding is detected in a 29-year-old man with a knife wound to the neck. During the initial treatment of the wound, it was established that a vessel located along the lateral edge of the sternoclavicular-mammoid m Identify this vessel:

A. carotis externa A. external carotid

V jugularis externa V external jugular

V. jugularis interna Internal jugular vein

V jugularis anterior 5 anterior jugular

A. carotis interna A. internal carotid

195 / 200
У хворого переливання крові ускладнилося розвитком гемотрансфузійного шоку. Назвіть тип алергічної реакції, що лежить в основі даної патології: The patient's blood transfusion was complicated by the development of hemotransfusion shock. Name the type of allergic reaction underlying this pathology:

Цитотоксичний Cytotoxic

Рецептороопосередкований Receptor Intermediation

Гіперчутливість сповільненого типу Delayed type hypersensitivity

Імунокомплексний Immunocomplex

Анафілактичний Anaphylactic

196 / 200
Підшлункова залоза - орган змішаної секреції. Ендокринно продукує бета-клітинами гормон інсулін, який впливає на обмін вуглеводів. Як він впливає на активність глікогенфо-сфорилази (ГФ) і глікогенсинтетази (ГС)? The pancreas is an organ of mixed secretion. Endocrinely, beta cells produce the hormone insulin, which affects carbohydrate metabolism. How does it affect the activity of glycogen phosphorylase (GF) and glycogen synthetase (GS)?

Пригнічує ГФ і ГС Suppresses GF and GS

Не впливає на активність ГФ і ГС Does not affect the activity of HF and HS

Активує ГФ, пригнічує ГС Activates HF, suppresses HS

Активує ГФ і ГС Activates GF and GS

Пригнічує ГФ, активує ГС Suppresses HF, activates HS

197 / 200
Призначення доксицикліну гідрохлориду викликало порушення симбіозу мікробної флори в кишечнику. Визначити тип порушень при антибіотикотерапії: The appointment of doxycycline hydrochloride caused a violation of the symbiosis of the microbial flora in the intestines. Determine the type of violations during antibiotic therapy:

Бактеріоз Bacteriosis

Дисбактеріоз Dysbacteriosis

Суперінфекція Superinfection

Сенсибілізація Sensitization

Ідіосинкразія Idiosyncrasy

198 / 200
Для проведення анальгезії наркотичний анальгетик застосували з препаратом бензодіазепінового ряду. Який засіб використали для потенціювання анальгезії? To provide analgesia, a narcotic analgesic was used with a benzodiazepine drug. Which agent was used to potentiate analgesia?

Хлорпротіксен Chlorprothixene

Діазепам Diazepam

Імізин Imisin

Трифтазин Triftazin

Карбамазепін Carbamazepine

199 / 200
У хворого стенокардія. Який антиангінальний засіб протипоказаний для лікування, якщо в пацієнта алергія на йод? The patient has angina. Which antianginal drug is contraindicated for treatment if the patient is allergic to iodine?

Аміодарон Amiodarone

Верапаміл Verapamil

Дротаверин Drotaverine

Нітрогліцерин Nitroglycerin

Нітросорбід Nitrosorbide

200 / 200
Молода людина 25-ти років споживає надмірну кількість вуглеводів (600 г на добу), що перевищує її енергетичні потреби. Який процес буде активуватися в організмі людини у даному випадку? A 25-year-old young man consumes an excessive amount of carbohydrates (600 g per day), which exceeds his energy needs. What process will be activated in the human body in this case ?

Гліколіз Glycolysis

Ілюконеогенез Iluconeogenesis

Ліполіз Lipolysis

Окиснення жирних кислот Oxidation of fatty acids

Ліпогенез Lipogenesis